Sunteți pe pagina 1din 101

EBOOK – O privire asupra teoriei numerelor

ISBN 978-606-500-162-6

Copyright 2022 © Editura GIL


Toate  drepturile  asupra  prezentei  ediţii  aparţin  Editurii  GIL.  Nicio 
parte din aceastǎ lucrare nu se poate reproduce fǎrǎ acordul scris al Editurii
GIL. 

Acest ebook este protejat de legea drepturilor de autor. Fiecare PDF este
securizat în 28 de zone cu watermark invizibil (id comandă, e-mail) pentru
a nu putea fi distribuit pe alte căi virtuale. Sistemul este unul complex şi se
detectează automat persoana care a redistribuit cartea ilegal.

EDITURA GIL
www.gil.ro
tel: 0733 - 677 992
web: ebooks.gil.ro
e-mail: comenzi@gil.ro
Cuvânt ı̂nainte

Orice carte este o oglindă a autorului ei. Este un loc ı̂n care autorul ı̂şi pune
o fărâmă din sufletul său pentru a o transmite mai departe. ”O privire asupra
teoriei numerelor” este o ocazie de a o cunoaşte pe Ana Cı̂rjea, o tânără matem-
aticiană foarte talentată, muncitoare şi perseverentă.

Este o carte pe care am văzut-o crescând de la dimensiunile unui manuscris


de câteva pagini la ceea ce este acum şi m-a bucurat să urmăresc ı̂n acelaşi timp
şi evoluţia Anei. Cartea conţine peste treizeci de probleme originale, dintre care
unele au fost deja publicate ı̂n ”Gazeta matematică”, toate foarte interesante. De
asemenea, o binevenită prezentare a unor idei teoretice, precum şi un număr de
probleme rezolvate, alese din diferite concursuri internaţionale.

Este o carte utilă oricărui elev sau student care doreşte să ştie mai multe ı̂n
domeniul teoriei numerelor, motiv pentru care o recomand cu căldură.

Mihai Chiş

3
Cuprins

1 Grupuri. Mulţimea numerelor naturale N şi mulţimea numerelor ı̂ntregi


Z 7

2 Inele şi corpuri 11

3 Relaţia de ordine pe N. Principiile inducţiei matematice 15

4 Divizibilitate. Algoritmul lui Euclid 17

5 Morfisme. Izomorfisme. Subgrupuri 21

6 Funcţii aritmetice 25

7 Congruenţe. Teorema lui Lagrange. Teorema chineză a resturilor 27

8 Clase de resturi 29

9 Teoremele lui Wilson şi Fermat. Teorema lui Euler 31

10 Ecuaţii diofantiene clasice 33


10.1 Ecuaţii diofantiene liniare . . . . . . . . . . . . . . . . . . . . . . . . 33
10.2 Triplete pitagoreice . . . . . . . . . . . . . . . . . . . . . . . . . . . . 34
10.3 Exemple de ecuaţii diofantiene pătratice . . . . . . . . . . . . . . . . 35
10.4 Exemple de ecuaţii diofantiene de grad superior . . . . . . . . . . . 36

5
6

11 Lifting the Exponent Lemma (LTE) 37

12 Ecuaţii de tip Pell 39

13 Simbolul lui Legendre 41

14 Secvenţe Farey 43

15 Secvenţe Lucas 45

16 Alte teoreme 47

17 Probleme rezolvate 49

18 Probleme propuse 63

19 Soluţiile problemelor propuse 71


Capitolul 1

Grupuri. Mulţimea numerelor


naturale N şi mulţimea numerelor
ı̂ntregi Z

Definiţia 1.1: Fie M o mulţime nevidă. Orice aplicaţie ϕ : M × M → M se


numeşte lege de compoziţie pe mulţimea M .

Definiţia 1.2: Fie M o mulţime nevidă. Dacă ” · ” : M × M → M este o lege de


compoziţie pe M cu proprietatea că (x · y) · z = x · (y · z), ∀x, y, z ∈ M, atunci
legea de compoziţie ” · ” este asociativă.

Definiţia 1.3: O pereche (S, ·) formată dintr-o mulţime nevidă S şi o lege de
compoziţie ” · ” care este asociativă se numeşte semigrup.

Definiţia 1.4: Fie M o mulţime nevidă. Dacă ” · ” : M × M → M este o lege


de compoziţie pe M cu proprietatea că x · y = y · x, ∀x, y ∈ M, atunci legea de
compoziţie ” · ” este comutativă.

7
8 Grupuri. Mulţimea numerelor naturale N şi mulţimea numerelor ı̂ntregi Z

Definiţia 1.5: Dacă legea de compoziţie dintr-un semigrup (S, ·) este şi comuta-
tivă, semigrupul se numeşte semigrup comutativ sau abelian.

Definiţia 1.6: Fie M o mulţime nevidă. Dacă ” · ” : M × M → M este o lege


de compoziţie pe M cu proprietatea că există un element e ∈ M astfel ı̂ncât
x · e = e · x = x, ∀x ∈ M, atunci se spune ca legea de compoziţie ” · ” admite
element neutru, iar e este elementul neutru admis.

Teorema 1.1: Fie M o mulţime nevidă. Dacă legea de compoziţie ” · ” : M × M →


M admite un element neutru, acesta este unic.

Definiţia 1.7: Dacă legea de compoziţie dintr-un semigrup (M, ·) admite un ele-
ment neutru, atunci semigrupul se numeşte monoid.

Definiţia 1.8: Dacă legea de compoziţie dintr-un monoid (M, ·) este şi comuta-
tivă, monoidul se numeşte monoid comutativ sau abelian.

Definiţia 1.9: Fie M o mulţime nevidă şi ” · ” : M × M → M o lege de compoziţie


pe M care admite elementul neutru e. Dacă x este un element din mulţimea M
pentru care există x′ ∈ M astfel ı̂ncât x · x′ = x′ · x = e, atunci x este simetrizabil,
iar elementul x′ ∈ M se numeşte simetricul elementului x ı̂n raport cu legea de
compoziţie ” · ”.

Teorema 1.2: Fie M o mulţime nevidă şi ” · ” : M × M → M o lege de compoziţie


pe M, care este asociativă şi admite un element neutru. Atunci, dacă un element
x ∈ M are un simetric ı̂n raport cu ” · ”, acesta este unic.

Definiţia 1.10: Fie G o mulţime nevidă şi ” · ” : M × M → M o lege de compoziţie


pe G. Perechea (G, ·) se numeşte grup dacă sunt ı̂ndeplinite următoarele axiome:
(G1) : Axioma asociativităţii, (x · y) · z = x · (y · z), ∀x, y, z ∈ G
(G2) : Axioma elementului neutru, ∃e ∈ G astfel ı̂ncât x · e = e · x = x, ∀x ∈ G
Grupuri. Mulţimea numerelor naturale N şi mulţimea numerelor ı̂ntregi Z 9

(G3) : Axioma elementelor simetrizabile, ∀x ∈ G, ∃x′ ∈ G astfel ı̂ncat x · x′ =


x′ · x = e.

Definiţia 1.11: Dacă legea de compoziţie dintr-un grup (G, ·) este şi comutativă,
grupul se numeşte grup comutativ sau abelian.

Propoziţia 1.1: Definim N = {0, 1, 2, ..., n, ...} ca fiind mulţimea numerelor nat-
urale şi legea de compoziţie ” + ” : N × N → N ca fiind adunarea numerelor
naturale. Atunci, perechea (N, +) este un monoid comutativ cu proprietatea de
simplificare.

Propoziţia 1.2: Definim legea de compoziţie ” · ” : N × N → N ca fiind ı̂nmulţirea


numerelor naturale. Atunci, perechea (N, ·) este un monoid comutativ.

Propoziţia 1.3: Definim Z = {..., −n, ..., −2, −1, 0, 1, 2, ..., n, ...} , unde n ∈ N, ca
fiind mulţimea numerelor ı̂ntregi şi legea de compoziţie ”+” : Z×Z → Z ca fiind
adunarea numerelor ı̂ntregi. Atunci, perechea (Z, +) este un grup comutativ.

Propoziţia 1.4: Definim legea de compoziţie ” · ” : Z × Z → Z ca fiind ı̂nmulţirea


numerelor ı̂ntregi. Atunci, perechea (Z, ·) este un monoid comutativ.
Capitolul 2

Inele şi corpuri

Definiţia 2.1: Fie A o mulţime nevidă şi legile de compoziţie:



 A × A → A, (x, y) → x⊥y
A × A → A, (x, y) → x ⊺ y

Tripletul (A, ⊥, ⊺) se numeşte inel dacă sunt ı̂ndeplinite următoarele axiome:


(A1) : Axioma grupului, perechea (A, ⊥) este un grup comutativ
(A2) : Axioma monoidului, perechea (A, ⊺) este monoid
(A3) : Axiomele distributivităţii,

 x ⊺ (y⊥z) = (x ⊺ y)⊥(x ⊺ z), ∀x, y, z ∈ A
(x⊥y) ⊺ z = (x ⊺ z)⊥(y ⊺ z), ∀x, y, z ∈ A.

Definiţia 2.2: Dacă legea de compoziţie ⊺ este şi comutativă, atunci inelul (A, ⊥, ⊺)
se numeşte inel comutativ.

Definiţia 2.3: Grupul (A, ⊥) se numeşte grupul subiacent al inelului (A, ⊥, ⊺).

Definiţia 2.4: Pentru simplificarea scrierii, atunci când este posibil, pentru cele
două legi de compoziţie ”⊥” şi ” ⊺ ” se folosesc notaţiile ” + ” şi ” · ”. Atunci,

11
12 Inele şi corpuri

prima operaţie a inelului se numeşte adunarea inelului, iar a doua operaţie se


numeşte ı̂nmulţirea inelului.

Observaţia 2.1: Cu notaţiile ” + ”, respectiv ” · ” pentru operaţiile inelului, ax-


iomele distributivităţii reprezintă reguli de ı̂nmulţire a unui element cu o sumă,
respectiv a ı̂nmultirii unei sume cu un element al inelului:

 x · (y + z) = x · y + x · z, ∀x, y, z ∈ A
(x + y) · z = x · z + y · z, ∀x, y, z ∈ A.

Definiţia 2.5: Elementul neutru al adunării inelului se numeşte element nul sau
zero şi se notează cu 0A sau 0.

Definiţia 2.6: Simetricul unui element x ∈ A ı̂n grupul subiacent (A, +) se


numeşte opusul lui x şi notează cu −x.

Definiţia 2.7: Dacă x, y ∈ A, elementul x + (−y), notat cu x − y, se numeşte


diferenţa elementelor x şi y.

Definiţia 2.8: Elementul neutru al monoidului (A, ·) se numeşte elementul uni-


tate al inelului şi se notează cu 1A sau 1.

Definiţia 2.9: Elementele simetrizabile ale monoidului (A, ·) se numesc elemente


inversabile ale inelului A şi se notează U(A). Atunci, perechea (U(A), ·) este un
grup numit grupul unităţilor inelului A. Dacă x este inversabil, inversul sau se
notează cu x−1 .

Teorema 2.1: Fie (A, +, ·) un inel nenul. Atunci, oricare ar fi x ∈ A, au loc


relaţiile: x · 0 = 0 · x = 0.

Definiţia 2.10: Fie (A, ⊥, ⊺) un inel cu elementul nul 0A . Un element d ∈ A \ {0A }


se numeşte divizor al lui zero dacă există d′ ∈ A \ {0A } astfel ı̂ncât d ⊺ d′ = 0A
sau d′ ⊺ d = 0A .
Inele şi corpuri 13

Definiţia 2.11: Un inel comutativ nenul şi fără divizori ai lui zero se numeşte
domeniu de integritate sau inel integru.

Teorema 2.2: Fie (A, +, ·) un inel. Atunci:



(−a) · b = a · (−b) = −(ab), ∀a, b ∈ A
 (−a) · (−b) = ab, ∀a, b ∈ A
(regula semnelor).

Teorema 2.3: Fie (A, +, ·) un inel integru, a ∈ A \ {0} şi x, y ∈ A. Dacă ax = ay,
atunci x = y (legea de simplificare la stânga). Dacă xa = ya, atunci x = y (legea
de simplificare la dreapta).

Definiţia 2.12: Un inel nenul (K, +, ·) ı̂n care orice element nenul este inversabil
se numeşte corp.

Definiţia 2.13: Dacă inelul K este comutativ, corpul K se numeşte corp comuta-
tiv.

Observaţia 2.2: Pentru un corp (K, +, ·) există egalitatea: U(K) = K \ {0} = K ∗ .


Atunci, perechea (K ∗ , ·) este un grup, numit grupul multiplicativ al corpului
K.

Observaţia 2.3: Inelele (C, +, ·), (R, +, ·) şi (Q, +, ·) sunt corpuri numite corpuri
numerice.
Capitolul 3

Relaţia de ordine pe N. Principiile


inducţiei matematice

Definiţia 3.1: Pentru două numere m, n ∈ N, spunem că:


a) m precede n sau că m este mai mic decât n şi scriem m < n dacă există
p ∈ N \ {0} astfel ı̂ncât m + p = n; se mai spune că n este (strict) mai mare decât
m (n > m);
b) m precede sau este egal cu n sau că m este mai mic sau egal decât n şi scriem
m ≤ n dacă există p ∈ N astfel ı̂ncât m + p = n; se mai spune că n este mai mare
sau egal decât m (n ≥ m).

Teorema 3.1: Pentru două numere m, n ∈ N are loc una singură dintre următoarele
trei relaţii: m < n, m = n sau m > n (Principiul trihotomiei).

Teorema 3.2: (N, ≤) este o mulţime bine ordonată (orice sumbulţime nevidă M
a lui N admite un prim element m ∈ M astfel ı̂ncât m ≤ x, ∀x ∈ M (Principiul
bunei ordonări).

Teorema 3.3: Fie P (n) o propoziţie ce poate fi asociată cu orice număr natural
n. Dacă P (0) este adevarată şi pentru un număr arbitrar m ∈ N avem că P (m)

15
16 Relaţia de ordine pe N. Principiile inducţiei matematice

adevarată implică P (m + 1) adevarată, atunci P (n) este adevarată pentru orice


n ∈ N (Principiul I al inducţiei matematice).

Teorema 3.4: Fie P (n) o propoziţie ce poate fi asociată cu orice număr natural
n. Dacă P (0) este adevarată şi pentru un număr arbitrar m ∈ N avem ca P (r)
adevarată pentru orice r < m, r ∈ N, implică P (m) adevarată, atunci P (n) este
adevarată pentru orice număr n ∈ N (Principiul II al inducţiei matematice).
Capitolul 4

Divizibilitate. Algoritmul lui Euclid

Teorema 4.1: Fie m, n ∈ N∗ , definită că N \ {0} . Atunci există şi sunt unice
q, r ∈ N astfel ı̂ncât: n = m · q + r, iar 0 ≤ r ≤ m − 1.

Definiţia 4.1: În relaţia de mai sus, q se numeşte câtul ı̂mpărţirii lui n la m , iar
r se numeşte restul ı̂mpărţirii lui n la m.

Definiţia 4.2: Fie n ∈ Z şi m ∈ N∗ . Dacă există q ∈ Z∗ , definită ca Z \ {0} , astfel


ı̂ncât n = m · q, atunci spunem că numărul n este divizibil cu m . Se notează
.
n..m.

Observaţia 4.1: Dacă n este divizibil cu m, se spune că m ı̂l divide pe n şi se
notează: m|n.

Definiţia 4.3: Dacă m ı̂l divide pe n, spunem că m este un divizor al lui n, iar n
este un multiplu al lui m.

Definiţia 4.4: Dacă n este un număr natural, atunci 1 şi n sunt divizori ai lui n
numiţi divizori improprii.

17
18 Divizibilitate. Algoritmul lui Euclid

Definiţia 4.5: Orice divizor al lui n diferit de divizorii improprii se numeşte divi-
zor propriu.

Definiţia 4.6: Definim mulţimea numerelor prime ca fiind mulţimea numerelor


naturale nenule p care nu au alţi divizori decât pe 1 şi pe ele ı̂nsele.

Teorema 4.2: Există o infinitate de numere prime (formulată de Euclid).

Conjectura 4.1: Oricare ar fi n ≥ 2 un număr natural, există un număr prim p


astfel ı̂ncât n < p < 2n. (Conjectura lui Bertrand).

Definiţia 4.7: Fie n, m ∈ N. Atunci, cel mai mare divizor comun al lui n şi m,
notat cu gcd(n, m), este definit ca fiind cel mai mare număr natural nenul care ı̂l
divide atât pe n, cât şi pe m.

Definiţia 4.8: Numerele naturale n şi m sunt prime ı̂ntre ele dacă gcd(n, m) = 1.

Definiţia 4.9: Fie n, m ∈ N. Atunci, cel mai mic multiplu comun al lui n şi m,
notat cu lcm(n, m), este definit ca fiind cel mai mic număr natural nenul care este
divizibil atât cu n, cât şi cu m.

Teorema 4.3: Prin ı̂nlocuirea repetată a numerelor naturale nenule m şi n cu


|n − m| = max {m, n} − min {m, n} , respectiv cu min {m, n} se poate obţine
gcd(m, n). (Algoritmul lui Euclid, varianta cu scăderi repetate).

Teorema 4.4: Prin ı̂nlocuirea repetată a numerelor naturale nenule m şi n, m ≥ n,


cu n, respectiv cu r, restul ı̂mpărţirii lui m la n, se poate obţine gcd(m, n).
(Algoritmul lui Euclid, varianta cu ı̂mpărţiri repetate).

Corolarul 4.1: Oricare ar fi a, b ∈ N∗ , există x, y ∈ Z astfel ı̂ncât ax+by = gcd(a, b).


(Identitatea lui Bezout - enunţată pentru prima dată de Claude-Gaspard Bachet de
Meziriac). Numărul gcd(a, b) este cel mai mic număr ı̂ntreg pozitiv pentru care
x, y pot fi găsiţi.
Divizibilitate. Algoritmul lui Euclid 19

Definiţia 4.10: Fie n ∈ N. Atunci, se defineşte puterea a n− a unui număr x sau


x la puterea n astfel: xn = x
| · x {z
· . . . · x} .
nori

Observaţia 4.2: Dacă m, n ∈ N, atunci au loc:


am · an = am+n , a ∈ R
am : an = am−n , a ∈ R∗
(am )n = amn , a ∈ R
am · b m = (ab)m , a, b ∈ R
 a m
am : b m = , a, b ∈ R, b ̸= 0.
b
Observaţia 4.3: x0 = 1, oricare ar fi x ∈ R∗ , iar 00 nu este definit.

Corolarul 4.2: Pentru a, m, n ∈ N şi a > 1 este adevarată relaţia:

gcd(am − 1, an − 1) = agcd(m,n) − 1.

Teorema 4.5: Fie x un număr natural, x ≥ 2. Atunci, orice număr natural nenul
n poate fi scris ı̂n mod unic sub forma:

n = ak xk + ak−1 xk−1 + ... + a1 x + a0 ,

unde ai , pentru orice i ∈ {0, 1, ..., k} , este un număr natural strict mai mic ca x,
iar ak ̸= 0.

Corolarul 4.3: Orice număr natural nenul poate fi scris ca sumă de puteri distincte
ale lui 2.

Definiţia 4.11: Fie n ∈ Z. Dacă n este divizibil cu 2, n este numit număr par.
Altfel, n este un număr impar.

Conjectura 4.2: Fie funcţia T : N∗ → N∗ , definită prin:



 n/2, 2 | n
T (n) =
(3n + 1)/2, 2 ∤ n.
20 Divizibilitate. Algoritmul lui Euclid

Atunci, oricare ar fi n ∈ N∗ , există k ∈ N astfel ı̂ncât T (T (. . . T (n) . . .)) = 1.


| {z }
kori
(Conjectura lui Collatz).

Definiţia 4.12: Factorialul unui număr natural nenul n este dat de produsul tu-
turor numerelor naturale nenule mai mici sau egale cu n : n! = 1 · 2 · . . . · n.

Observaţia 4.4: Prin convenţie, 0! = 1.

Definiţia 4.13: O extindere Cantor a unui număr natural nenul n este o sumă:
n = ak k! + ak−1 (k − 1)! + . . . + a1 1!, unde ai ∈ {0, 1, . . . , i} , pentru orice i ∈
{1, 2, . . . , k} .
Capitolul 5

Morfisme. Izomorfisme. Subgrupuri

Definiţia 5.1: Fie (G1 , ·) şi (G2 , ∗) două grupuri. Fie f : G1 → G2 o funcţie. Dacă
f (x · y) = f (x) ∗ f (y), ∀x, y ∈ G1 , atunci f este un morfism (omomorfism) de
grupuri.

Definiţia 5.2: Fie (G1 , ·) şi (G2 , ∗) două grupuri. Fie f : G1 → G2 o funcţie. Dacă
f este un morfism de grupuri şi este bijectivă, atunci f este un izomorfism de
grupuri. În acest caz, grupurile (G1 , ·) şi (G2 , ∗) se numesc grupuri izomorfe.

Teorema 5.1: Fie (G1 , ·) şi (G2 , ∗) două grupuri cu elementele neutre e1 , respectiv
e2 . Fie f : G1 → G2 un morfism de grupuri. Atunci:



 f (e1 ) = e2

f (x−1 ) = (f (x))−1 , ∀x ∈ G1


 f (xn ) = (f (x))n , ∀x ∈ G , n ∈ Z.

1

Definiţia 5.3: Fie (G, ·) un grup. Atunci, un morfism f : G → G se numeşte


endomorfism, iar un izomorfism f : G → G se numeşte automorfism. Mulţimea
endomorfismelor unui grup G se notează cu End(G), iar mulţimea automorfis-
melor lui G se notează cu Aut(G).

21
22 Morfisme. Izomorfisme. Subgrupuri

Teorema 5.2: Fie (G, ·) un grup. Atunci, (End(G), ◦) este monoid şi (Aut(G), ◦)
este grup.

Definiţia 5.4: Fie (G, ·) un grup. Dacă H este o submulţime nevidă a lui G, iar
perechea (H, ·) formează un grup, atunci H este un subgrup al lui G.

Definiţia 5.5: Dacă (G.·) este un grup cu elementul neutru e, atunci ({e} , ·) şi
(G, ·) sunt subgrupuri ale lui G numite subgrupuri improprii.

Definiţia 5.6: Orice subgrup al lui (G, ·) diferit de subgrupurile improprii se


numeşte subgrup propriu.

Teorema 5.3: Fie (G, ·) un grup şi (H, ·) un subgrup al acestuia. Dacă e ∈ G şi
e′ ∈ H sunt elementele neutre ı̂n G şi H, atunci e = e′ . Dacă x ∈ H, iar x′ si x′′
sunt simetricele lui x ı̂n G, respectiv H, atunci x′ = x′′ .

Teorema 5.4: Fie (G, ·) un grup şi H o submulţime nevidă a lui G. Atunci, H
este un subgrup al lui G dacă şi numai dacă: ∀x, y ∈ H, xy −1 ∈ H, dacă şi numai
dacă H este parte stabilă a lui G(∀x ∈ H ⇒ x−1 ∈ H).

Teorema 5.5: Fie H ⊂ Z o mulţime nevidă. Atunci, H este subgrup al grupului


(Z, +) dacă şi numai dacă există n ∈ N astfel ı̂ncât H = nZ = {nx|x ∈ Z} .

Definiţia 5.7: Fie (G, ·) un grup. Dacă mulţimea G este finită, atunci grupul
(G, ·) se numeşte grup finit (analog se definesc inelul şi corpul finit). În acest caz,
ordinul grupului (G, ·) este notat cu ord(n) şi este egal cu numărul de elemente
ale mulţimii G.

Teorema 5.6: Orice corp finit este comutativ. (Teorema lui Wedderburn).

Teorema 5.7: Fie (G, ·) un grup finit, H un subgrup al lui G şi x, y ∈ G. Atunci, a)
mulţimile H şi xH au acelaşi număr de elemente; b) xH = yH sau xH ∩yH = ∅.
Morfisme. Izomorfisme. Subgrupuri 23

Teorema 5.8: Fie (G, ·) un grup finit şi H un subgrup al lui G. Atunci,
a) ord(H) | ord(G);
b) ord(G) = ord(H) · ord(G \ H)
(Teorema lui Lagrange).

Teorema 5.9: Fie (G, ·) un grup finit şi p un număr prim astfel ı̂ncât p|ord(G).
Atunci, numărul soluţiilor ecuaţiei xp = 1 este un multiplu nenul al lui p (Teorema
lui Cauchy).

Definiţia 5.8: Fie (A, ◦, ∗) si (B, ⊥, ⊺) două inele. O funcţie f : A → B se numeşte


morfism
 de inele dacă:
 f (x ◦ y) = f (x)⊥f (y), ∀x, y ∈ A


f (x ∗ y) = f (x) ⊺ f (y), ∀x, y ∈ A


f (1A ) = 1B .

Definiţia 5.9: Funcţia f : A → B se numeşte izomorfism de inele dacă este


morfism de inele şi dacă este bijectivă. În acest caz, inelele (A, ◦, ∗) şi (B, ⊥, ⊺) se
numesc inele izomorfe.

Observaţia 5.1: Un morfism de inele f : A → B este ı̂n particular un morfism de


grupuri. Astfel,



 f (0A ) = 0B
f (−x) = −f (x), ∀x ∈ A


f (nx) = n · f (x), ∀x ∈ A, n ∈ Z.

Observaţia 5.2: Dacă inelele (A, +, ·) şi (B, +, ·) sunt izomorfe, atunci grupurile
(A, +) şi (B, +) sunt izomorfe.

Definiţia 5.10: Fie (A, +, ·) un inel. Atunci, un morfism f : A → A se numeşte


endomorfism, iar un izomorfism f : A → A se numeşte automorfism. Mulţimea
endomorfismelor unui inel A se notează cu End(A), iar mulţimea automorfis-
melor lui A se notează cu Aut(A).
24 Morfisme. Izomorfisme. Subgrupuri

Definiţia 5.11: Fie (K, +, ·) şi (K ′ , +, ·) două corpuri. Dacă funcţia f : K → K ′


este morfism (izomorfism) de inele de la K la K ′ , atunci aceasta se numeşte
morfism (izomorfism) de corpuri.

Teorema 5.10: Orice morfism de corpuri este funcţie injectivă.


Capitolul 6

Funcţii aritmetice

Definiţia 6.1: O funcţie care este definită pentru toate numerele naturale nenule
se numeşte funcţie aritmetică.

Definiţia 6.2: O funcţie aritmetică f este multiplicativă dacă şi numai dacă f (mn) =
f (m)f (n), unde m şi n sunt numere naturale nenule prime ı̂ntre ele.

Teorema 6.1: Orice număr n ∈ N, n ≥ 2, poate fi scris ı̂n mod unic sub forma:
n = pα1 1 pα2 2 ...pαk k , unde p1 , p2 , ..., pk sunt numere prime distincte, iar α1 , α2 , ..., αk
sunt numere naturale nenule.

Definiţia 6.3: Scrierea lui n sub forma n = pα1 1 pα2 2 ...pαk k se numeşte descom-
punerea lui n ı̂n factori primi.

Teorema 6.2: Funcţia lui Euler (numită şi indicatorul lui Euler), notată cu φ, se
defineşte astfel: φ(n), unde n ∈ N, este egal cu numărul de numere naturale
mai mici ca sau egale cu n care sunt prime cu n. Atunci, pentru n = pα1 1 pα2 2 ...pαk k
descompunerea
 lui
 n ı̂n factori
  primi, are
 loc relaţia:
1 1 1
φ(n) = n 1 − 1− ... 1 − .
p1 p2 pk

25
26 Funcţii aritmetice

Teorema 6.3: Pentru n = pα1 1 pα2 2 ...pαk k descompunerea lui n ı̂n factori primi,
numărul divizorilor naturali ai lui n este: τ (n) = (α1 + 1)(α2 + 1)...(αk + 1).

Teorema 6.4: Pentru n = pα1 1 pα2 2 ...pαk k descompunerea lui n ı̂n factori primi,
pα1 +1 − 1 pα2 2 +1 − 1
suma tuturor divizorilor pozitivi ai lui n este: σ(n) = 1 · ·
p1 − 1 p2 − 1
pαk +1 − 1
... · k .
pk − 1

Teorema 6.5: Funcţiile aritmetice φ, τ şi σ sunt funcţii multiplicative.

Teorema 6.6: Dacă k, n sunt numere naturale nenule, atunci:


X n · φ(n)
k= .
2
1≤k≤n,gcd(k,n)=1

Teorema 6.7: Dacă p este un număr prim, atunci: φ(p) = p − (τ (n))2 + 3.

Definiţia 6.4: Funcţia lui Mobius este o funcţie definită astfel: µ : N∗ → Z,





 1, dacă n = 1
µ(n) = 0, dacă există p prim, p2 |n


(−1)k , n = p1 p2 . . . pk , pi prime distincte.

Teorema 6.8: Fie f : N∗ → C o funcţie aritmetică. Dacă definim F : N∗ → C ca


X
F (n) = f (d), are loc identitatea:
d|n
X n X n
f (n) = F (d) · µ = µ(d) · F .
d d
d|n d|n
(Teorema de inversiune a lui Mobius).
Capitolul 7

Congruenţe. Teorema lui Lagrange.


Teorema chineză a resturilor

.
Definiţia 7.1: Dacă a, b ∈ Z, n ∈ N∗ , astfel ı̂ncât a − b..n, atunci spunem că a este
congruent cu b modulo n şi se notează a ≡ b( mod n).

Propoziţia 7.1: Fie n ≥ 2 un număr natural şi a un număr ı̂ntreg prim cu n.


Atunci, ecuaţia ax ≡ b( mod n) are o soluţie unică modulo n : x = b · aφ(n)−1 .

Corolarul 7.1: Fie n ≥ 2 un număr natural şi a un număr ı̂ntreg, astfel ı̂ncât
gcd(a, n) = d. Atunci, ecuaţia ax ≡ b( mod n) are soluţii dacă şi numai dacă d|b,
caz ı̂n care numărul soluţiilor este chiar d.

Teorema 7.1: Dacă a, n ∈ N astfel ı̂ncât gcd(a, n) = 1, atunci congruenţa ax ≡


1( mod n) are o soluţie x = a−1 ∈ Z. Numărul x se numeşte inversul lui a(
mod n). Soluţia generală a congruenţei este dată de toate numerele x ∈ Z cu
proprietatea că x ≡ a−1 ( mod n).

27
28 Congruenţe. Teorema lui Lagrange. Teorema chineză a resturilor

Teorema 7.2: Fie P (X) = an X n + . . . + a1 X + a0 un polinom şi p un număr prim


astfel ı̂ncât p ∤ an . Atunci, congruenţa P (x) ≡ 0( mod p) are cel mult n soluţii.
(Teorema lui Lagrange).

Definiţia 7.2: Fie a, n două numere naturale nenule, prime ı̂ntre ele, astfel ı̂ncât
n ≥ 2. Atunci, cel mai mic număr natural nenul k pentru care ak ≡ 1( mod n),
notat cu γn (a), se numeşte ordinul lui a modulo n sau ordinul gaussian.

Definiţia 7.3: Fie n ≥ 2 un număr natural şi a un număr ı̂ntreg prim cu n. Dacă
ordn (a) = φ(a), atunci a se numeşte rădăcina primitivă modulo n.

Teorema 7.3: Fie m1 ,  m2 , ..., mn ∈ N, prime ı̂ntre ele două câte două. Atunci,


 x ≡ a1 ( mod m1 )

 x ≡ a ( mod m )

2 2
sistemul de n ecuaţii:


 ...


x ≡ a ( mod m )
n n
are o soluţie unică pentru x modulo m1 m2 ...mn (Teorema chineză a resturilor).
Capitolul 8

Clase de resturi

Teorema 8.1: Fie n ∈ N∗ şi Rn = {0, 1, 2, ..., n − 1} mulţimea resturilor obţinute la


ı̂mpărţirea numerelor ı̂ntregi prin n. Dacă pe mulţimea Rn se definesc operaţiile
de adunare şi ı̂nmulţire modulo n:

” ⊕ ” : R × R → R , a ⊕ b ≡ a + b( mod n)
n n n
,
 ” ⊙ ” : Rn × Rn → Rn , a ⊙ b ≡ a · b( mod n)

atunci (Rn , ⊕) este un grup abelian, iar (Rn , ⊙) este un monoid abelian.

Definiţia 8.1: Fie n ∈ N∗ . Dacă a ∈ Z, folosim notaţia b


a pentru mulţimea tuturor
numerelor ı̂ntregi de forma ns + a, cu s ∈ Z, numită clasa de resturi modulo n
de reprezentant a.
n o
Teorema 8.2: Fie n ∈ N∗ şi Zn = b
0, b
1, b − 1 mulţimea claselor de resturi
2, ..., n[
modulo n. Dacă pe mulţimea Zn se definesc operaţiile de adunare şi ı̂nmulţire a
” + ” : Z × Z → Z , b
n a+b = a⊕b
n n
b [
claselor de resturi modulo n: ,
 ” · ” : Zn × Zn → Zn , b ⊙b
a + bb = a[
atunci (Zn , ⊕) este un grup abelian numit grupul aditiv al claselor de resturi
modulo n, iar (Zn , ⊙) este un monoid abelian.

29
30 Clase de resturi
n o
Pentru U(Zn ) = k̂ ∈ Zn |(n, k) = 1 , (U(Zn ), ·) este un grup abelian, numit grupul
multiplicativ al claselor de resturi modulo n.

Teorema 8.3: Fie n ∈ N∗ si x


b ∈ Zn . Atunci, clasa de resturi x
b este un divizor al
lui zero dacă şi numai dacă gcd(x, n) = d > 1.

Teorema 8.4: Inelul (Zn , +, ·) este un corp dacă şi numai dacă n este număr prim.

Observaţia 8.1: Dacă p este un număr prim, atunci există corpuri cu p elemente.
Un astfel de corp este Zp . Orice corp finit are pn elemente, unde p este un număr
prim.
Capitolul 9

Teoremele lui Wilson şi Fermat.


Teorema lui Euler

Lema 9.1: Fie p un număr prim şi 0 < k < p, k natural. Atunci, k 2 ≡ 1(
mod p) ⇔ k = 1 sau k = p − 1.

Teorema 9.1: Fie p ∈ N, p ≥ 2. Atunci, p este prim ⇔ (p − 1)! ≡ −1( mod p)


(Teorema lui Wilson).

Teorema 9.2: Fie p un număr prim şi a un număr ı̂ntreg astfel ı̂ncât gcd(a, p) = 1.
Atunci, ap−1 ≡ 1( mod p) (Mica Teoremă a lui Fermat).

Teorema 9.3: Dacă n, a ∈ N∗ astfel ı̂ncât gcd(a, n) = 1, atunci aφ(n) ≡ 1( mod n)


(Teorema lui Euler).

31
Capitolul 10

Ecuaţii diofantiene clasice

10.1 Ecuaţii diofantiene liniare

Definiţia 10.1.1: O ecuaţie de forma a1 x1 + a2 x2 + ... + an xn = b se numeşte


ecuaţie diofantiană liniară, unde a1 , a2 , ..., an sunt numere ı̂ntregi fixate, numite
coeficienţi, iar n ∈ N∗ şi b ∈ Z sunt şi ei fixaţi.

Teorema 10.1.1: Ecuaţia precedentă se poate rezolva ⇔ gcd(a1 , a2 , ..., an )|b. În
caz de solvabilitate, soluţiile ı̂ntregi ale ecuaţiei se exprimă ı̂n funcţie de n − 1
parametri ı̂ntregi.

Corolarul 10.1.1: Fie a1 , a2 ∈ Z astfel ı̂ncât:


gcd(a1 , a2 ) = 1. Dacă (x01 , x02 ) este o soluţie a ecuaţiei a1 x1 + a2 x2 = b, atunci
toate
 soluţiile ei sunt date de:
x = x + a t
1 01 2
, unde t ∈ Z.
x2 = x02 − a1 t

33
34 Ecuaţii diofantiene clasice

10.2 Triplete pitagoreice

Teorema
 10.2.1: Soluţiile ecuaţiei a2 + b2 = c2 , unde a, b, c ∈ Z, sunt date de:
2 2
a = t(m − n )


b = t · 2mn , unde t, m, n ∈ Z.


c = t(m2 + n2 )

Atunci, tripletele (a, b, c) se numesc triplete pitagoreice.

Teorema 10.2.2: Generalizarea imediată a ecuaţiei pitagoreice iniţiale este dată de


ecuaţia: x2 +y
2
+z 2 = t2 , x, y, z, t ∈ Z. Toate soluţiile ı̂n N ale ecuaţiei generalizate


 x = 2l


 y = 2m


sunt date de: l2 + m2 − n2 pentru x, y pare,
 z =

 n
l2 + m2 + n2



t =

n
unde l, m ∈ N sunt arbitrare, iar n este un divizor al lui l2 + m2 mai mic decât

l2 + m2 .
Ecuaţii diofantiene clasice 35

10.3 Exemple de ecuaţii diofantiene pătratice

Teorema
 10.3.1: Toate soluţiile ı̂ntregi ale ecuaţiei x2 + axy + y 2 = z 2 sunt date
{x, y} = {l(an2 − 2mn), l(m2 − n2 )}
de: ,
 z = l(amn − m2 − n2 )
unde l, m, n ∈ Z.

Teorema 10.3.2: Toate soluţiile ı̂n N ale ecuaţiei x4 + x2 y 2 + y 4 = z 2 sunt date de:
(x, y, z) ∈ {(n, 0, n2 ), (0, n, n2 )} , unde n ∈ N.

Teorema 10.3.3: Toate soluţiile ı̂n N ale ecuaţiei x4 − x2 y 2 + y 4 = z 2 sunt date de:
(x, y, z) ∈ {(n, 0, n2 ), (0, n, n2 ), (n, n, n2 )} , unde n ∈ N.
36 Ecuaţii diofantiene clasice

10.4 Exemple de ecuaţii diofantiene de grad superior

Teorema 10.4.1: Ecuaţia x4 + y 4 = z 2 nu are soluţii ı̂n Z∗ .

Teorema 10.4.2: Ecuaţia x4 − y 4 = z 2 nu are soluţii ı̂n Z∗ .

Teorema 10.4.3: Ecuaţia xn + y n = z n nu are soluţii ı̂n Z∗ dacă n > 2, n ∈ N


(Marea Teoremă a lui Fermat).

Conjectura 10.4.1: Singura soluţie a ecuaţiei xp − y q = ±1 ı̂n Z \ {0, 1} este


32 − 23 = 1 (Conjectura lui Catalan).
Capitolul 11

Lifting the Exponent Lemma (LTE)

Definiţia 11.1: Definim vp (n) = k ca fiind un număr natural cu proprietatea că


pk |n, dar pk+1 ∤ n, unde p este un număr prim, iar n un număr ı̂ntreg.

Lema 11.1: Fie x, y ∈ Z şi n ∈ N∗ . Dat fiind un număr prim arbitrar p ̸= 2 astfel
ı̂ncât p|x − y, dar p ∤ x, p ∤ y, avem că: vp (xn − y n ) = vp (x − y) + vp (n) (Prima formă
a LTE).

Lema 11.2: Fie x, y ∈ Z şi n un număr natural impar. Dat fiind un număr prim
arbitrar p ̸= 2 astfel ı̂ncât p|x + y, dar p ∤ x, p ∤ y, avem că: vp (xn + y n ) =
vp (x + y) + vp (n) (A doua formă a LTE).

Teorema 11.1: Fie x, y două numere naturale impare şi n ∈ N∗ , n par. Atunci,
v2 (xn − y n ) = v2 (x − y) + v2 (x + y) + v2 (n) − 1.

37
Capitolul 12

Ecuaţii de tip Pell

Definiţia 12.1: Fie D ∈ N astfel ı̂ncât D nu este pătrat perfect. Atunci, o ecuaţie
de forma x2 − Dy 2 = 1, cu x, y ∈ Z, este o ecuaţie Pell.

Teorema 12.1: Dacă (x0 , y0 ) este cea mai mică soluţie a ecuaţiei Pell x2 − Dy 2 = 1,
din N × N şi diferită de (1, 0), atunci toate soluţiile ı̂ntregi (x, y) ale ecuaţiei sunt
√ √
date de relaţia: x + y D = ±(x0 + y0 D)n , astfel soluţia generală (xn , yn ) fiind
dată de (xn , yn )n≥0 , definit prin:

xn+1 = x0 xn + Dy0 yn


yn+1 = y0 xn + x0 yn


x1 = x0 , y1 = y0 .

Definiţia 12.2: Fie D ∈ N. Atunci, o ecuaţie de forma x2 − Dy 2 = −1, cu x, y ∈ Z,


este o ecuaţie Pell negativă.

Observaţia 12.1: Spre deosebire de ecuaţia Pell x2 − Dy 2 = 1, care este solvabilă


pentru orice D ≥ 2 care nu este pătrat perfect, ecuaţia Pell negativă este solvabilă
numai pentru anumite valori ale lui D.

Teorema 12.2: Dacă p este un număr prim astfel ı̂ncât p ≡ 1( mod 4), atunci
ecuaţia Pell negativă x2 − py 2 = −1 este solvabilă ı̂n N∗ .

39
40 Ecuaţii de tip Pell

Teorema 12.3: În cazul ı̂n care ecuaţia Pell negativă x2 − Dy 2 = −1 este sol-
vabilă ı̂n N∗ , având cea mai mică soluţie (x0 , y0 ), soluţia generală este dată de

 n , yn )n≥0 , definit prin:


(x
x = y u + Dx v
n 0 n 0 n
,
 y n = x0 xn + y 0 y n
unde (un , vn )n≥0 este soluţia ecuaţiei Pell u2 − Dv 2 = 1.
Capitolul 13

Simbolul lui Legendre

Definiţia 13.1: Fie a ∈ Z şi p un număr prim. Dacă există x ∈ Z astfel ı̂ncât
x2 ≡ a( mod p), atunci a este rest pătratic modulo p.

Definiţia 13.2: Simbolul lui Legendre pentru un număr ı̂ntreg a şi un număr
prim p este:

  
a 1, dacă a este rest pătratic ( mod p) şi p ∤ a

= 0, dacă p|a
p 

−1, altfel.

Teorema 13.1: Dacă p ̸= 2 este un număr prim, atunci jumătate dintre elementele
mulţimii {1, 2, ..., p − 1} sunt resturi pătratice.

Teorema
  13.2: Dacă p ̸= 2 este un număr prim şi a ∈ N∗ nedivizibil cu p, atunci:
a p−1
≡ a 2 ( mod p) (Criteriul lui Euler).
p
 
ab
Teorema 13.3: Dacă p ̸= 2 este un număr prim şi a, b ∈ Z, atunci: =
   p
a b
.
p p

41
42 Simbolul lui Legendre
 
−1
Teorema 13.4: Dacă p ̸= 2 este un număr prim, atunci: = 1 ⇔ p ≡ 1(
p
mod 4)

Lema 13.1: Fie p ̸= 2 un număr prim şi a ∈ Z astfel ı̂ncât gcd(p, a) = 1. Cele
mai mici resturi modulo p sunt {0, 1, ..., p − 1} . Considerăm cele mai mici resturi
pozitive modulo p ale numerelor a,2a,3a, ..., p−1

2
a . Dacă notăm cu N numărul
p a
resturilor mai mari decât , atunci: = (−1)N (Lema lui Gauss).
2 p

Teorema 13.5: Fie p ̸= 2 şi a un număr impar nedivizibil cu p. Atunci,


P p−1
 
a a
= (−1) i=1 [i· p ] .
2

Teorema 13.6: Fie p, q ∈ P \ {2} , p ̸= q. Atunci,


  
p q p−1 q−1
= (−1) 2 · 2 .
q p

(Legea de reciprocitate a lui Gauss)


Capitolul 14

Secvenţe Farey

Definiţia 14.1: Pentru orice număr n ∈ N∗ , secvenţa Farey Fn este şirul de nu-
a a
mere raţionale (dacă a, b ∈ Z, b ̸= 0, atunci ∈ Q, numită mulţimea nu-
b b
merelor raţionale) cu 0 ≤ a ≤ b ≤ n şi gcd(a, b) = 1, ordonat crescător.
a c
Teorema 14.1: Dacă < , unde a, b, c, d ∈ N, b, d nenule, atunci
b d
a a+c c
< < ,
b b+d d
a+c
unde este medianta celor două fracţii.
b+d
a c a
Teorema 14.2: Fie , ∈ Q astfel ı̂ncât 0 ≤ ≤ cd ≤ 1 şi bc − ad = 1. Atunci,
b d b
a c
şi sunt termeni consecutivi ı̂n secvenţele Farey Fn care au proprietatea că:
b d
max {b, d} ≤ n ≤ b + d − 1.
a c a e
Teorema 14.3: Fie , ∈ Q astfel ı̂ncât 0 ≤ ≤ cd ≤ 1 şi bc − ad = 1. Dacă
b d b f
a c
este medianta fracţiilor şi , atunci:
b d
a e c
a) < < ;
b f d
b) be − af = 1;
c) cf − de = 1.

43
Capitolul 15

Secvenţe Lucas

Definiţia 15.1: Fie P, Q două numere ı̂ntregi astfel ı̂ncât D ≡ P 2 − 4Q > 0. Fie
1 √ 1 √
a, b rădăcinile ecuaţiei x2 − P x + Q = 0, a ≡ (P + D), b ≡ (P − D), astfel
2 2
1 2 √ an − b n
ı̂ncât a + b = P, ab = (P − D) = Q, a − b = D. Atunci, Un (P, Q) ≡ şi
4 a−b
Vn (P, Q) ≡ an + bn , care satisfac relaţiile de recurenţă:

am+n − bm+n (am − bm )(an + bn ) an bn (am−n − bm−n )


Um+n = = − =
a−b a−b a−b
= Um Vn − an bn Um−n ,

Vm+n = am+n + bm+n = (am + bm )(an + bn ) − an bn (am−n + bm−n ) =


= Vm Vn − an bn Vm−n

se numesc secvenţele Lucas.

Definiţia 15.2: Numerele Lucas sunt numerele ce alcătuiesc secvenţa (Ln )n≥1 ,
definită prin recurenţa liniară: Ln = Ln−1 + Ln−2 , cu L1 = 1, L2 = 3.

45
46 Secvenţe Lucas

Teorema 15.1: Forma generală a elementelor din S, irul lui Lucas este:
√ !n √ !n
1+ 5 1− 5
Ln = + .
2 2
(formula lui Binet a numerelor lui Lucas).

Definiţia 15.3: Numerele Fibonacci sunt numerele ce alcătuiesc secvenţa (Fn )n≥1 ,
definită prin recurenţa liniară: Fn = Fn−1 + Fn−2 , cu F1 = F2 = 1. Prin convenţie,
F0 = 0.

Teorema 15.2: Forma generală a elementelor din S, irul lui Fibonacci este:
√ √
(1 + 5)n − (1 − 5)n
Fn = √ .
2n 5
(formula lui Binet a numerelor lui Fibonacci).

Definiţia 15.4: Secţiunea de aur este numărul ϕ, care notat sub forma unei fracţii
continue este:
1
ϕ=1+ 1 ,
1+ 1+ 1
..
1+ .


q p
fiind scris şi sub forma: ϕ = 1 + 1 + 1 + . . . şi având valoarea fixă ϕ =

5+1
.
2
ϕ+1
Propoziţia 15.1: Numărul ϕ ı̂ndeplineşte proprietatea: = ϕ, astfel ϕ2 − ϕ −
ϕ
1 = 0.

X (−1)n+1
Propoziţia 15.2: Cu notaţiile uzuale, ϕ = 1 + .
n=1
Fn Fn+1
 n
n ϕ
Propoziţia 15.3: Ca funcţii ale lui ϕ, Ln = [ϕ ], iar Fn = √ , unde [x]
5
reprezintă numărul ı̂ntreg cel mai apropiat de x.

Observaţia 15.1: Numerele Lucas şi numerele Fibonacci sunt secvenţe Lucas
Vn (1, −1), respectiv Un (1, −1).
Capitolul 16

Alte teoreme

Teorema 16.1: Dacă a, b, n sunt numere naturale nenule astfel ı̂ncât a > b, gcd(a, b) =
1 şi n ≥ 2, atunci an − bn are cel puţin un factor prim care nu ı̂l divide pe ak − bk
pentru orice număr natural nenul k < n, cu excepţia cazului ı̂n care a = 2, b = 1
şi n = 6 şi a cazului ı̂n care n = 2 şi a + b este o putere a lui 2.
(Prima parte a teoremei lui Zsigmondy)

Teorema 16.2: Dacă a, b, n sunt numere naturale nenule astfel ı̂ncât a > b şi
n ≥ 2, atunci an + bn are cel puţin un factor prim care nu ı̂l divide pe ak + bk
pentru orice număr natural nenul k < n, cu excepţia cazului ı̂n care a = 2, b = 1
şi n = 3.
(A doua parte a teoremei lui Zsigmondy)

Teorema 16.3: Dacă p este un număr prim, p > 3, atunci numărătorul lui Hp−1 =
1 1 1 1
1 + + ... + este divizibil cu p2 , iar numărătorul lui Hp−1,2 = 1 + 2 + 2 +
2 p−1 2 3
1
... + este divizibil cu p. Acestea implică faptul că
(p − 1)2
 
2p − 1
≡ 1( mod p3 ),
p−1

47
48 Alte teoreme
 
n n!
unde = este un coeficient binomial.
p (n − p)!p!
(Teorema lui Wolstenholme)
Capitolul 17

Probleme rezolvate

1. Găsiţi toate numerele prime p astfel ı̂ncât şi 2p + p2 este tot prim.
sursa - Problema 56

S OLUŢIE: Dacă p > 3, p este impar şi 3 ∤ p. Atunci, 2p ≡ (−1)p ≡ −1 ≡ 2( mod 3)


şi p2 ≡ (±1)2 ≡ 1( mod 3), astfel 2p + p2 ≡ 0( mod 3), dar 2p + p2 ≥ 22 + 22 > 3,
deci nu poate fi prim. Dacă p = 2, 2p + p2 = 22 + 22 = 8, care nu este prim. Dacă
p = 3, 2p + p2 = 23 + 32 = 8 + 9 = 17, cum 17 este prim rezultă că unicul număr
prim care ı̂ndeplineşte această condiţie este p = 3.

2. Să se rezolve ı̂n N ecuaţia (x + y)2 + 3x + y + 1 = z 2 .


sursa - L. Panaitopol, A.Gica, Probleme de aritmetică şi teoria numerelor. Idei şi
metode de rezolvare

S OLUŢIE: Notăm x + y = s, x − y = d.
Atunci, s2 + 2s + d + 1 = z 2 , (s + 1)2 + d = z 2 .
Dacă d ≥ 0, d = (z − s − 1)(z + s + 1), unde z − s − 1 = a, z + s + 1 = b, b >
a+b b−a b−a−2
a ≥ 0.d = ab, z = ,s = −1 = .s ≥ d, deci b − a − 2 ≥ 2ab.
2 2 2
Dacă a ≥ 1, b − a − 2 ≥ 2ab ≥ 2b ≥ b > b − a − 2, contradicţie. Atunci, a = 0, d =

49
50 Probleme rezolvate

0, x = y şi s + 1 = 2x + 1 = z. Pentru soluţia (x, y, z) = (n, n, 2n + 1), are loc:


(x + y)2 + 3x + y + 1 = 4n2 + 4n + 1 = (2n + 1)2 = z 2 .
Dacă d < 0, fie D = −d, D > 0, D = (s + 1 − z)(s + 1 + z), unde s + 1 − z =
b−a a+b b+a−2
a, s + 1 + z = b, b > a ≥ 0.D = ab, z = ,s = −1 = .s ≥ D, deci
2 2 2
b + a − 2 ≥ 2ab. Dacă a ≥ 1, b + a − 2 ≥ ab + ab ≥ b + a > b + a − 2, contradicţie.
Atunci, a = 0, D = 0, dar D > 0, nu convine.
Soluţia rămâne (x, y, z) = (n, n, 2n + 1), n ∈ N.

3. Demonstraţi că există o infinitate de perechi de numere ı̂ntregi pozitive relativ


prime a, b > 1 pentru care a + b divide ab + ba .
sursa - USAMO 2017/1, propusă de Gregory Galperin

dd + 2d
S OLUŢIE: Se consideră construcţia d ≡ 1( mod 4), d > 1, şi x = şi luăm
d+2
x+d x−d
a= ,b = .
2 2
Pentru verificare, se verifică ı̂n primul rând că b este impar, iar a este par. Fie
d = a − b, d impar. Atunci, a + b|ab + ba ⇔ (−b)b + ba ≡ 0( mod a + b) ⇔
⇔ ba−b ≡ 1( mod a + b) ⇔ bd ≡ 1( mod d + 2b) ⇔ (−2)d ≡ dd ( mod d + 2b) ⇔
⇔ d + 2b|dd + 2d .
dd + 2 d 1 dd + 2d
 
Ar fi suficient ca d + 2b = ⇒b= − d , care este ceea ce am
d+2 2 d+2
construit. Totodată, cum gcd(x, d) = 1 rezultă că gcd(a, b) = gcd(d, b) = 1.
sursa - USAMO 2017 Notes, Evan Chen

4. Găsiţi toate numerele ı̂ntregi pozitive n astfel ı̂ncât restul ı̂mpărţirii lui nn la
n2 + 1 este un pătrat perfect.
sursa - MNC 2019/1
Probleme rezolvate 51

S OLUŢIE: Dacă n ≡ 0( mod 4), atunci nn = (n2 + 1)(nn−2 − nn−4 + nn−6 − nn−8 +
... + n2 − 1) + 1. Cum 1 este pătrat perfect, n = 4k, k ∈ N∗ este soluţie.
Dacă n ≡ 1( mod 4), atunci nn = (n2 +1)(nn−2 −nn−4 +nn−6 −nn−8 +...+n3 −n)+n.
Cum 1 este singurul rest modulo 4 pe care ı̂l au pătratele perfecte impare, n =
(2k + 1)2 , k ∈ N este soluţie.
Dacă n ≡ 2( mod 4), atunci nn = (n2 + 1)(nn−2 − nn−4 + nn−6 − nn−8 + ... + n4 −
n2 ) + n2 . Cum n2 este un pătrat perfect, n = 4k + 2, k ∈ N este soluţie.
Dacă n ≡ 3( mod 4), atunci nn = (n2 + 1)(nn−2 − nn−4 + nn−6 − nn−8 + ... + n5 −
n3 + n − 1) + n2 − n + 1. Cum n2 − 2n + 1 < n2 − n + 1 < n2 , (n − 1)2 < n2 − n + 1 <
n2 , n2 − n + 1 nu poate fi ı̂n acest caz pătrat perfect.
Astfel, numerele căutate sunt toate numerele naturale pare nenule şi pătratele
perfecte ale numerelor impare.

5. Găsiţi toate numerele ı̂ntregi pozitive x, y, z astfel ı̂ncât 45x − 6y = 2019z .


sursa - JBMO Shortlist 2019 N5
S OLUŢIE: x, y > 0, deci u(45x ) = 5, u(6y ) = 6, u(45x − 6y ) = u(15 − 6) = 9,
dar u(2019z ) ∈ {9, 1} , deci u(2019z ) = u(45x − 6y ) = 9, z este impar, astfel
2019z ≡ (−1)z ≡ −1( mod 4), −1 ≡ 45x − 6y ≡ 1x − 6y ≡ 1 − 6y ( mod 4), deci
6y ≡ 2( mod 4). Dacă y ≥ 2, 6y ≡ 0( mod 4), iar y ∈ N∗ , deci y = 1.
45x = 6 + 2019z . Dacă z ≥ 2, 45x ≡ 6 + 2019z ≡ 6 + 0 ≡ 6( mod 9), dar x ≥ 1,
deci 45x ≡ 0( mod 9), contradicţie. Atunci, z = 1, 45x = 6 + 2019 = 2025 = 452 ,
deci (x, y, z) = (2, 1, 1).

6. Găsiţi toate numerele m, n ∈ N şi numerele prime p ≥ 5 care satisfac:

m(4m2 + m + 12) = 3(pn − 1).

sursa - INMO 2013, Problem 2


52 Probleme rezolvate

S OLUŢIE: 4m3 + m2 + 12m + 3 = 3pn


⇒ 4m(m2 + 3) + m2 + 3 = 3pn
⇒ (4m + 1)(m2 + 3) = 3pn .
Fie d ∈ N, d = gcd(4m + 1, m2 + 3). Atunci:

d|4m + 1, d|m2 + 3 ⇒ d|4m2 + m, d|4m2 + 12


⇒ d|m − 12
⇒ d|4m − 48, d|4m + 1
⇒ d|49
⇒ d ∈ {1, 7, 49}

. . . . .
Dacă d..7, (4m + 1)(m2 + 3)..7, 3pn ..7 ⇒ pn ..7, p..7, deci p = 7.
.
i) d = 7. Dacă m2 + 3..3, m2 + 3 = 3 · 7a si 4m + 1 = 7b cu a + b = n, a, b ∈ N.
. . .
Dacă a, b ≥ 2, m2 + 3..49, 4m + 1..49, deci d..49, nu convine. Dacă a sau b = 0,
atunci m2 + 3 sau 4m + 1 nu este divizibil cu 7, nu convine. Atunci, a sau b = 1.
Pentru a = 1, m2 + 3 = 21, m2 = 18, deci m nu este natural, nu convine. Pentru
b = 1, 4m + 1 = 7, 4m = 6, nici ı̂n acest caz m nu este natural.
.
Dacă 4m + 1..3, m2 + 3 = 7a şi 4m + 1 = 3 · 7b cu a + b = n, a, b ∈ N. Dacă
. . .
a, b ≥ 2, m2 + 3..49, 4m + 1..49, deci d..49, nu convine. Dacă a sau b = 0, atunci
m2 + 3 sau 4m + 1 nu este divizibil cu 7, nu convine. Atunci, a sau b = 1. Pentru
a = 1, m2 + 3 = 7, m2 = 4, deci m = 2, 4m + 1 = 9, care nu este divizibil cu 7.
Pentru b = 1, 4m + 1 = 21, m = 5, deci 7a = 28, a nu este ı̂n N.
.
ii) d = 49. Dacă m2 + 3..3, m2 + 3 = 3 · 7a şi 4m + 1 = 7b cu a + b = n, a, b ∈ N. Dacă
. . .
a, b ≥ 3, m2 + 3..343, 4m + 1..343, deci d..343, nu convine. Dacă a sau b ≤ 1, atunci
m2 +3 sau 4m+1 nu este divizibil cu 49, nu convine. Atunci, a sau b = 2. Pentru
a = 2, m2 + 3 = 147, m = 12. Atunci, 7b = 49, b = a = 2, deci (m, n, p) = (12, 4, 7).
Analog dacă b = 2.
Probleme rezolvate 53

.
Dacă 4m + 1..3, m2 + 3 = 7a şi 4m + 1 = 3 · 7b cu a + b = n, a, b ∈ N. Dacă
.. . .
a, b ≥ 3, m2 + 3.343, 4m + 1..343, deci d..343, nu convine. Dacă a sau b ≤ 1,
atunci m2 + 3 sau 4m + 1 nu este divizibil cu 49, nu convine. Atunci, a sau
b = 2. Pentru a = 2, m2 + 3 = 49, m2 = 46 care nu este pătrat perfect. Pentru
b = 2, 4m + 1 = 147, 2m = 73, care este impar.
Dacă d = 1, 4m + 1 sau m2 + 3 ∈ {1, 3} . Dacă 4m + 1 = 1, m = 0, m2 + 3 = 3,
deci 3 · pn = 3, pn = 1, dar p ≥ 5, deci n = 0, (m, n, p) = (0, 0, p), unde p este un
număr prim, p ≥ 5. Dacă 4m + 1 = 3, m nu este ı̂n N. Dacă m2 + 3 = 1, m2 < 0,
iar dacă m2 + 3 = 3 atunci 4m + 1 = 1, este acelaşi caz.

7. Găsiţi toate numerele ı̂ntregi a, b, c cu 1 < a < b < c pentru care (a − 1)(b −
1)(c − 1) este un divizor al lui abc − 1.
sursa - IMO 1992, Day 1, Problem 1

S OLUŢIE: Fie a − 1 = r, b − 1 = s, c − 1 = t. Atunci, 0 < r < s < t şi rst|(r +


1)(s + 1)(t + 1) − 1
⇒ rst|rst + rs + st + tr + r + s + t
⇒ rst|rs + st + tr + r + s + t
⇒ există k ∈ N∗ astfel ı̂ncât k · rst = rs + st + tr + r + s + t.
Dacă k ≥ 3, k · rst ≥ 3rst = 3rst − r − s − t + r + s + t > 3rst − 2rs − rt + r + s + t =
rs(t − 2) + st · r + tr(s − 1) + r + s + t ≥ rs + st + tr + r + s + t = k · rst, contradicţie.
Rezultă că k ≤ 2.
i) k = 1
⇒ rst = rs + st + tr + r + s + t
⇒ t(rs − r − s − 1) = rs + r + s
⇒ rs − r − s − 1|rs + r + s
⇒ rs − r − s − 1|2r + 2s + 1
⇒ există l ∈ N∗ astfel ı̂ncât l|rs − r − s − 1| = 2r + 2s + 1.
54 Probleme rezolvate

Dacă r ≥ 4, rs − r − s − 1 ≥ 4s − r − s − 1 = 3s − r − 1 > 2s − 1 > 0, deci


l · (rs − r − s − 1) = 2r + 2s + 1. Pentru l ≥ 6, l(rs − r − s − 1) ≥ 6rs − 6r − 6s − 6 >
24s − 6s − 6s − 6 = 12s − 6 > 2r + 2s + 8 − 6 > 2r + 2s + 1 = l · (rs − r − s − 1),
contradicţie, deci l ≤ 5. De asemenea, cum 2r + 2s + 1 este impar, se observă că şi
l este impar. Dacă l = 5, 5rs−7s = 7r +6, s(5r −7) = 7r +6 ≥ 5(5r −7) = 25r −35,
deci 41 ≥ 18r > 3 · 18 = 54 > 41, contradicţie. Dacă l = 3, s(3r − 5) = 5r + 4 ≥
5(3r−5) = 15r−25, 29 ≥ 10r > 30 > 29, contradicţie. Dacă l = 1, s(r−3) = 3r+2,
deci r − 3|3r + 2, r − 3|3r − 9 ⇒ r − 3|11, r ∈ {4, 14} . Pentru r = 4, s = 14, atunci
rs + r + s 74
t= = = 2 < r < t, contradicţie. Pentru r = 14, s = 4 < r < s,
rs − r − s − 1 37
contradicţie.
Dacă r = 3, 3s − 3 − s − 1|6 + 2s + 1, 2s − 4|2s + 7, 2|2s − 4, deci 2|2s + 7, nu
convine.
Dacă r = 2, 2s−2−s−1|4+2s+1, s−3|2s+5, s−3|2s−6, deci s−3|11, s ∈ {4, 14} .
rs + r + s 14
Dacă s = 4, t = = = 14. Atunci, (a, b, c) = (3, 5, 15). Dacă
rs − r − s − 1 1
rs + r + s 44
s = 14, t = = = 4 < s < t, contradicţie.
rs − r − s − 1 11
Dacă r = 1, s − 1 − s − 1|2 + 2s + 1, 2|2s + 3, nu convine.
ii) k = 2
⇒ 2rst = rs + st + tr + r + s + t
⇒ t(2rs − r − s − 1) = rs + r + s
⇒ 2rs − r − s − 1|rs + r + s
⇒ 2rs − r − s − 1|2rs + 2r + 2s,
2rs − r − s − 1|2rs − r − s − 1
⇒ 2rs − r − s − 1|3r + 3s + 1
⇒ 0 ≤ r − 1 ≤ r(s − 1) + s(r − 1) − 1 =
= 2rs − r − s − 1 ≤ 3r + 3s + 1
⇒ 2rs ≤ 4r + 4s + 2, rs ≤ 2r + 2s + 1.
Dacă r ≥ 4, 2rs ≥ 4s + 5s > 4r + 4s + 2, nu convine.
Dacă r = 3, 5s − 4|3s + 10
Probleme rezolvate 55

⇒ 5s − 4|15s + 50, 5s − 4|15s − 12


⇒ 5s − 4|62
⇒ 5s − 4 ∈ {1, 2, 31, 62}
⇒ 5s ∈ {5, 6, 35, 66}
⇒ s ∈ {1, 7} ⇒ s = 7, 31t = 31, t = 1 < r < t.
Dacă r = 2, 3s − 3|3s + 7, 3|7, contradicţie.
Dacă r = 1, s − 2|3s + 4
⇒ s − 2|3s − 6, s − 2|3s + 4
⇒ s − 2|10, s − 2 ∈ {1, 2, 5, 10}
⇒ s ∈ {3, 4, 7, 12} .
Dacă s = 3, t = 7 ⇒ (a, b, c) = (2, 4, 8). Dacă s = 4, 2t = 9, nu convine. Dacă
s = 7, 5t = 15, t = 3 < s. Dacă s = 12, 10t = 25, nu convine.
Atunci, soluţiile sunt: (a, b, c) ∈ {(3, 5, 15), (2, 4, 8)} .

8. Găsiţi toate cvintupletele de numere ı̂ntregi pozitive (a, n, p, q, r) astfel ı̂ncât


an − 1 = (ap − 1)(aq − 1)(ar − 1).
sursa - Japanese MO 2011

S OLUŢIE: Dacă a ≥ 3, (ap − 1)(aq − 1)(ar − 1) > amax{p,q,r} − 1, astfel n > p, n > q
şi n > r. Dacă n ≥ 3, din teorema lui Zsigmondy rezultă că există un factor
prim s al lui an − 1 astfel ı̂ncât s ∤ (ap − 1)(aq − 1)(ar − 1), deci acest caz nu
convine. Dacă n < 3, cum p, q, r ≥ 1, n > p, q, r ⇒ n = 2, p = q = r = 1, de unde
a2 − 1 = (a − 1)3 , a2 − 1 = a3 − 3 · a2 + 3 · a − 1, a3 − 4 · a2 + 3 · a = 0, a ̸= 0, deci
a2 − 4a + 3 = 0, (a − 2)2 = 1, a − 2 = 1, a = 3, deci (a, n, p, q, r) = (3, 2, 1, 1, 1).
Dacă a < 3, a = 1 sau a = 2. Dacă a = 1, celelalte numere pot lua orice valoare:
(a, n, p, q, r) = (1, n, p, q, r). Dacă a = 2, n ≥ 2 şi n ̸= 6, atunci din nou se observă
că n = p, q sau r cu soluţia (a, n, p, q, r) = (2, 2 ≤ n ̸= 6, 2 ≤ n ̸= 6, 1, 1), (2, 2 ≤
n ̸= 6, 1, 2 ≤ n ̸= 6, 1) sau (2, 2 ≤ n ̸= 6, 1, 1, 2 ≤ n ̸= 6), sau n > p, q, r şi se poate
56 Probleme rezolvate

aplica teorema lui Zsigmondy, astfel neexistând soluţii ı̂n acest caz. Dacă n =
1, p = q = r = 1, (a, n, p, q, r) = (2, 1, 1, 1, 1). Dacă n = 6, (ap − 1)(aq − 1)(ar − 1) =
(2p −1)(2q −1)(2r −1) = 26 −1 = 63 = (2−1)(2−1)(26 −1) = (22 −1)(22 −1)(23 −1),
de unde (a, n, p, q, r) = (2, 6, 1, 1, 6), (2, 6, 1, 6, 1), (2, 6, 6, 1, 1),
(2, 6, 2, 2, 3), (2, 6, 2, 3, 2) sau (2, 6, 3, 2, 2).

9. Dacă p este un număr prim astfel ı̂ncât p ≡ 1( mod 4), atunci ecuaţia Pell
negativă x2 − py 2 = −1 este solvabilă ı̂n N∗ .
(teorema 10.2)

S OLUŢIE: Considerăm ecuaţia Pell u2 − pv 2 = 1 cu soluţia fundamentală (u0 , v0 ).


Atunci, u20 − 1 = (u0 + 1)(u0 − 1) = pv02 . Dacă u0 este par, atunci pv02 ≡ 3( mod 4),
atunci v02 ≡ 3( mod 4), astfel u0 este impar, gcd(u0 − 1, u0 + 1) = 2, u0 ± 1 =
2a2 p, u0 ∓ 1 = 2b2 , unde v0 = 2ab, gcd(a2 p, b2 ) = 1, ±1 = b2 − pa2 . Cum (u0 , v0 )
este soluţia fundamentală a ecuaţiei 1 = u2 − pv 2 , dacă 1 = b2 − pa2 , am avea
a ≥ v0 , dar a < v0 , deci −1 = b2 − pa2 , b2 − pa2 = −1, astfel ecuaţia x2 − py 2 = −1
este solvabilă ı̂n N∗ .
sursa - T. Andreescu, D. Andrica, O introducere ı̂n studiul ecuaţiilor diofantiene

10. Determinaţi toate tripletele (p, m, n) de numere ı̂ntregi pozitive astfel ı̂ncât p
este prim şi 2m p2 + 1 = n5 .
sursa - Problem 516

S OLUŢIE: 2m p2 = n5 −1 = (n−1)(n4 +n3 +n2 +n+1).gcd(n−1, n4 +n3 +n2 +n+1) =


gcd(n − 1, (n − 1)(n3 + 2n2 + 3n + 4) + 5) = gcd(n − 1, 5). Dacă n − 1 ≡ 0(
.
mod 5), 2m p2 ..5, de unde p = 5. Atunci, cum n4 + n3 + n2 + n + 1 = (n2 + 1) · n(n +
1)+1, fiind astfel impar, n−1 = 2m ·5, n4 +n3 +n2 +n+1 = 5, de unde n = 1, 2m = 0,
Probleme rezolvate 57

nu convine. Astfel, gcd(n − 1, 5) = gcd(n4 + n3 + n2 + n + 1, 5) = 1, p ̸= 5, iar


.
n − 1..2m , n4 + n3 + n2 + n + 1 > 1, deci n4 + n3 + n2 + n + 1 = p2 , n − 1 = 2m .
Atunci, p2 = (2m +1)4 +(2m +1)3 +(2m +1)2 +2m +1+1 = 24m +4·23m +6·22m +4·2m +
1+23m +3·22m +3·2m +1+22m +2·2m +1+2m +1+1 = 24m +5·23m +10·22m +10·2m +5.
Dacă m ≥ 2, p2 ≡ 5( mod 8), dar n2 ≡ 0, 1 sau 4( mod 8), ∀n ∈ N, deci p nu
există ı̂n aceste condiţii. Astfel, m < 2, m = 1.p2 = 16 + 5 · 8 + 10 · 4 + 10 · 2 + 5 =
16 + 40 + 40 + 20 + 5 = 121 = 112 , p = 11, n = 2m + 1 = 3, deci (p, m, n) = (11, 1, 3).

11. Pentru un număr prim p > 3 şi numere ı̂ntregi arbitrare a, b, demonstraţi că
abp − bap este divizibil cu 6p.
sursa - Problem 264

. .
S OLUŢIE: Dacă a..p sau b..p, atunci abp − bap ≡ ab(bp−1 − ap−1 ) ≡ 0( mod p).
Altfel, din mica teoremă a lui Fermat rezultă că bp−1 ≡ ap−1 ≡ 1( mod p), deci
abp − bap ≡ ab(bp−1 − ap−1 ) ≡ 0( mod p). Astfel, abp − bap este divizibil cu p.
Dacă a sau b este par, atunci abp − bap este par. Dacă a, b sunt impare, abp , bap
sunt impare, abp − bap este par. Atunci, abp − bap este divizibil cu 2.
.
Dacă a sau b este divizibil cu 3, atunci abp − bap ..3. Altfel, cum p − 1 este par,
bp−1 ≡ ap−1 ≡ 1( mod 3), deci abp −bap , fiind divizibil cu bp−1 −ap−1 , este divizibil
cu 3.
Astfel, cum gcd(2, 3) = gcd(2, p) = gcd(3, p) = 1, atunci abp − bap este divizibil cu
2 · 3 · p = 6p.

12. Demonstraţi că, dacă n > 1 este un număr ı̂ntreg, atunci n5 + n + 1 are cel
puţin doi divizori primi distincţi.
sursa - Problem 442
58 Probleme rezolvate

.
S OLUŢIE: Se observă (din cazuri n ≤ 4 ) că n5 + n + 1..n2 + n + 1. Avem că
n5 + n + 1 = (n2 + n + 1)(n3 − n2 + 1).gcd(n2 + n + 1, n3 − n2 + 1) = gcd(n3 + n2 +
n, n3 − n2 + 1) = gcd(2n2 + n − 1, n3 + n2 + n) = gcd(2n2 + n − 1, n2 + n + 1) =
gcd(n2 + n + 1, 2n2 + 2n + 2 − 2n2 − n + 1) = gcd(n2 + n + 1, n + 3) = gcd(n2 + 3n −
n2 − n − 1, n + 3) = gcd(2n − 1, n + 3) = gcd(2n + 6 − 2n + 1, n + 3) = gcd(7, n + 3).
Dacă 7 ∤ n+3, gcd(n2 +n+1, n3 −n2 +1) = 1, iar cum n2 +n+1 > 1, n3 −n2 +1 > 1,
rezultă că există p, q prime, p ̸= q, astfel p|n2 +n+1, q|n3 −n2 +1, deci p, q|n5 +n+1.
.
Altfel, gcd(n2 + n + 1, n3 − n2 + 1) = 7. Astfel, n5 + n + 1..7 (1).
Presupunem că n2 +n+ 1 = 7a , n3 −n2 + 1 = 7b . Cum gcd(n2 +n+ 1, n3 −n2 + 1) =
7 < 72 , rezultă ca a sau b = 1.
Dacă a = 1, n2 + n + 1 = 7, dar n2 + n + 1 ≥ 22 + 2 + 1 = 7, cu egalitate
⇔ n = 2. Atunci, n5 + n + 1 = 32 + 2 + 1 = 35, divizibil cu 5 şi 7. Dacă
b = 1, n3 −n2 +1 = 7, n2 (n−1) = 6. Dar n ≥ 2, iar, pentru n = 2, n2 (n−1) = 4 ̸= 6.
Pentru n ≥ 3, n2 (n − 1) ≥ 32 · 2 > 6. Atunci, pentru n = 2 se găsesc aceste 2
numere prime, iar pentru n ≥ 3 nu există a, b deci există un număr prim diferit
de 7 care ı̂l divide pe n5 + n + 1.
Din relaţia (1) rezultă concluzia.

13. Pentru a, m, n ∈ N şi a > 1 este adevarată relaţia: gcd(am − 1, an − 1) =


agcd(m,n) − 1.
(colorarul 4.2)

xk − 1 xnk − 1
S OLUŢIE: Folosind identitatea = xk−1 + ... + x + 1, avem că =
x−1 x−1
(x(n−1)k + x(n−2)k + ... + xk + 1)(xk−1 + ... + x + 1). Atunci, ax − 1|ay − 1 dacă x|y,
astfel agcd(m,n) − 1 divide atât am − 1, cât şi an − 1. Pe de altă parte, dacă r|am − 1
şi r|an − 1, atunci gcd(r, a) = 1, astfel a este inversabil mod r. Din Identitatea
lui Bezout, există x, y ∈ Z astfel ı̂ncât xn + ym = gcd(m, n). Atunci, agcd(m,n) ≡
Probleme rezolvate 59

axn+ym ≡ (an )x (am )y ≡ 1( mod r). Orice divizor comun al numerelor an − 1 şi
am − 1 este şi un divizor al lui agcd(m,n) − 1, astfel gcd(an − 1, am − 1) = agcd(m,n) − 1.
sursa - Exponent lifting, Yufei Zhao

14. Secvenţele {an } şi {bn } satisfac a0 = 1, b0 = 0 şi, pentru n = 0, 1, 2, ...,


an+1 = 7an + 6bn − 3,
bn+1 = 8an + 7bn − 4.
Demonstraţi că an este un pătrat perfect pentru toţi n = 0, 1, 2, ...
sursa - Problem 338

S OLUŢIE:
 
a
n+1 = 7a n + 6b n − 3 7a
n+1 = 49an + 42bn − 21
⇒ ⇒ 7an+1 −6bn+1 = an +3 ⇒
 bn+1 = 8an + 7bn − 4  6bn+1 = 48an + 42bn − 24
⇒ 6bn+1 = 7an+1 − an − 3 ⇒ 6bn = 7an − an−1 − 3, deci an+1 = 14an − an−1 − 6.
Atunci, 2an+1 = 14·2an −2an−1 −14+1+1 ⇒ (2an+1 −1) = 14·(2an −1)−(2an−1 −1).
Notând xn = 2an − 1 pentru n ≥ 0 ⇒ xn+1 = 14xn − xn−1 .
Atunci, secvenţa {xn } are ecuaţia caracteristică: t2 = 14t − 1 ⇒ t2 − 14t + 1 = 0.
2 2
Aici, ∆ = 14 √ − 4 = 192 = 8√ · 3, deci
14 ± 82 · 3 14 ± 8 3 √
t1,2 = = = 7 ± 4 3.
2 √ 2 √
⇒ xn = a · (7 + 4 3)n + b · (7 − 4 3)n , a, b fixate.

x0 = a+b = 2a0 −1 = 1, x1 = 7(a+b)+4 3(a−b) = 2a1 −1 = 2(7·1+6·0−3)−1 =
√ 1
= 7 = 7(a + b), deci 4 3(a − b) = 0, a − b = 0 ⇒ a = b = .
√ √ √ 2 √
(7 + 4 3)n + (7 − 4 3)n 2 + (7 + 4 3)n + (7 − 4 3)n
xn = ⇒ an = .
2 4
√ √
s s r r
p √ p √ 7 + 72 − 48 7 − 72 − 48 7+1 7−1
7 + 4 3 = 7 + 48 = + = + =
√ √ 2 2 2 2
4 + 3,
60 Probleme rezolvate

√ √
s s r r
p √ p √ 2
7 + 7 − 48 7 − 72 − 48 7+1 7−1
7 − 4 3 = 7 − 48 = − = − =
√ √ 2 2 2 2
4 − 3. √ √ √ √
( 4 + 3)2n + 2(4 − 3)n + ( 4 − 3)2n
Atunci, an = =
 √ n 2 √ n 4 √ n  √ n 2
(2 + 3) + 2 · (2 + 3) · (2 − 3) + (2 − 3)
= =
√ √ 4
! 2
(2 + 3)n + (2 − 3)n
= .
2
1 √ 1 √
Fie secvenţa {yn } , yn = · (2 + 3)n + · (2 − 3)n pentru n ≥ 0. Aceasta are
2 2 √ √
2 4±2 3 α± ∆
ecuaţia caracteristică t − αt + β = 0, cu t1,2 = = .
2 2
2
α = 4, ∆ = 12 = α − 4β = 16 − 4β, deci β = 1.
Ecuaţia devine t2 − 4t + 1 = 0, atunci: yn+1 − 4yn + yn−1 = 0, yn+1 = 4yn − yn−1 .
Pentru y0 = 1 şi y1 = 4 : 2 = 2, y0 , y1 ∈ Z, de unde yn ∈ Z pentru n = 0, 1, 2, ...,
astfel an = yn2 este pătrat perfect pentru n = 0, 1, 2, ...

15. Rezolvaţi următoarea ecuaţie ı̂n mulţimea numerelor naturale: x2 = 2y +


2021z .
sursa - Serbia JBMO, problema 2

S OLUŢIE:
x2 ≡ 2y + 2z ( mod 3). Dacă y ≡ z( mod 2), x2 ≡ 2( mod 3), dar x2 ≡ 0 sau 1(
mod 3), deci nu convine, y şi z au parităţi diferite.
Dacă y = 2b, z = 2c+1, b, c ∈ N, atunci x2 −22b = (x−2b )(x+2b ) = 2021z = 432c+1 ·
472c+1 .gcd(x + 2b , x − 2b ) = gcd(x + 2b , 2b+1 ). Dacă b = 0, x2 = 1 + 2021z , u(x2 ) = 2,
deci x2 nu poate fi pătrat perfect. Atunci, b ≥ 1, x2 este impar, x + 2b este impar,
deci gcd(x + 2b , 2b+1 ) = 1, atunci există două variante:

i) x + 2b = 472c+1 , x − 2b = 432c+1 ⇒ 2b+1 ≡ 472c+1 − 432c+1 ≡ (−1)2c+1 − 32c+1 ≡


−1 − 9c · 3 ≡ −1 − 3 · 1c ≡ −1 − 3 ≡ 4( mod 8), atunci 2b+1 = 4, b + 1 = 2, b =
Probleme rezolvate 61

1, ⇒ y = 2.472c+1 − 432c+1 = 4. Dacă c ≥ 1, 472c+1 − 432c+1 = (47 − 43)(472c +


472c−1 · 43 + . . . + 432c ) ≥ 4 · (472 + 47 · 43 + 432 ) > 4, nu convine. Atunci,
c = 0, 472c+1 − 432c+1 = 47 − 43 = 4.z = 1 şi x2 = 22 + 20211 = 2025 = 452 ,
deci (x, y, z) = (45, 2, 1).

ii) x + 2b = 20212c+1 , x − 2b = 1, 2b+1 ≡ 20212c+1 − 1 ≡ 52c+1 − 1 ≡ 25c · 5 − 1 ≡


1c · 5 − 1 ≡ 5 − 1 ≡ 4( mod 8), atunci 2b+1 = 4 = 22 , b + 1 = 2, b = 1 ⇒ y = 2, x =
1 + 21 = 3, 20212c+1 = 3 + 2 = 5, nu convine.
Dacă y = 2b + 1, z = 2c, b, c ∈ N, atunci x2 − 20212c = (x + 2021c )(x − 2021c ) =
22b+1 , gcd(x + 2021c , x − 2021c ) = gcd(x + 2021c , 2 · 2021c ).gcd(22b+1 ,
2021c ) = 1, x+2021c |22b+1 , astfel gcd(x+2021c , 2021c ) = 1, gcd(x+2021c , 2·2021c ) =
.
gcd(x + 2021c , 2). Cum 2b + 1 ≥ 1, 22b+1 ..2; cum x + 2021c ≡ x − 2021c ( mod 2), iar
. .
(x + 2021c )(x − 2021c ) = 22b+1 ..2, x + 2021c ..2. Atunci, gcd(x + 2021c , x − 2021c ) = 2,
dar x + 2021c = 2s , x − 2021c = 2t astfel ı̂ncât s + t = 2b + 1, gcd(2s , 2t ) = 2, deci
s = 1 sau t = 1, iar celălalt este mai mare sau egal ca 2.x + 2021c > x − 2021c , deci
s > t, t = 1, s = 2b. Atunci, x = 2021c +2, x2 = (2021c +2)2 = 20212c +4·2021c +4 =
20212c +22b+1 , 4·2021c +4 = 22b+1 , 22b−1 ≡ 2021c +1 ≡ 1c +1 ≡ 2( mod 4), 22b−1 = 2,
deci 2b − 1 = 1, b = 1.x + 2021c = 22b = 22 = 4 < 2021, deci c = 0, x = 3. Atunci,
x2 = 9 = 23 + 20210 , adevărat: (x, y, z) = (3, 3, 0).
Capitolul 18

Probleme propuse

În continuare am notat cu P mulţimea numerelor prime şi cu Mn orice multiplu


natural al lui n.

1. Fie funcţia f : N → N care ı̂ndeplineşte următoarele condiţii:


(i) f (f (m)) + f (n)|m + n, ∀m, n ∈ N, m + n ̸= 0;
(ii) f (m + n) ≤ f (f (m) + f (n)), ∀m, n ∈ N;
(iii) f este crescătoare.
Arătaţi că f (m)|m, ∀m ∈ N∗ .

2. Determinaţi toate funcţiile f : N∗ → N∗ care ı̂ndeplinesc proprietăţiile:


(i) f (1) este un număr prim;
(ii) f (a) · b|f 2 (b) · a;
(iii) a · f (b)|f (a) · b2 .

63
64 Probleme propuse

3. Determinaţi toate funcţiile f : N∗ → N∗ care ı̂ndeplinesc proprietăţiile:


(i) pa |f (a);
(ii) f 2 (b)f (a)|p2a p2b a2 b2 , unde pa este al a-lea număr prim, a ∈ N∗ .

4. Rezolvaţi ı̂n N3 ecuaţia: 5x + 3y = 2z , ştiind că z ≡ y + 1( mod 4).

5. Determinaţi p ∈ P şi n ∈ N astfel ı̂ncât: 13p−1 + p3 = n2 .

6. Demonstraţi că nu există p prim şi n natural nenul astfel ı̂ncat pn + 3p = np .

1
7. Fie p un număr prim fixat şi funcţia f : M → Q definită prin: f (n) = pn+p ,
φ(n)
unde M = {n ∈ N|∀q ∈ N∗ , q|n, atunciq ≥ p}. Demonstraţi că:
 2p
n+m 1
f (n)f (m) ≤ p · 1+ ,
p−1

unde n, m ∈ M astfel ı̂ncât n are k divizori primi distincţi, respectiv m are l, iar
k + l = 2p.

8. Rezolvaţi ı̂n N3 ecuaţia: 5m + 7n = 6p .


Probleme propuse 65

9. Fie m,n două numere naturale nenule care ı̂ndeplinesc:


h m i2
(i) m + n|m · + n;
n h m i 2
(ii) m + n|gcd(m, n) · +1 .
n
Demonstraţi că n este un divizor al lui m (s-a notat cu [x] partea ı̂ntreaga a numă-
rului x ).

b b
10. Rezolvaţi ı̂n N2 ecuaţia: ab·a = ba·b .


k
" √ 3 #
φ(n) p Y pi − 1
11. Demonstraţi că: · τ (n) ≥ 2pi · , ∀n ∈ N, n ≥ 3, unde
τ (n) i=1
pi
p1 , p2 , ..., pk sunt toţi divizorii primi, distincţi, ai lui n.

12. Rezolvaţi ı̂n N2 ecuaţia: x5 + x7 = 4y · 10.

13. Determinaţi p, q ∈ P, distincte, astfel ı̂ncât există m, n ∈ N∗ , m ≥ n, pentru


care p2n − q m = (2n + m)2 .

14. Găsiţi perechile de numere naturale nenule (m, n) care ı̂ndeplinesc relaţia
hni
nm · m! = (m + 1)n · (n + 1), dacă m ≥ (s-a notat cu [x] partea ı̂ntreagă a
2
numărului x ).


66 Probleme propuse

15. Fie n un număr natural nenul, n ≥ 2. Dacă n are k divizori primi distincţi,
atunci arătaţi că:
τ (nx ) ≤ (x + 1)τ (n) , ∀x ∈ R+

16. Demonstraţi că există un număr finit de perechi de numere naturale (m, n),
unde n nu este un pătrat perfect, care sunt soluţii ale ecuaţiei
√ 3
[ n] = 2m · n (s-a notat cu [x] partea ı̂ntreagă a numărului x ).

17. Determinaţi funcţiile periodice f : N∗ → N, de perioadă k ∈ P, care ı̂ndeplinesc:


(i) 1 + f (n)|n2 , ∀n ∈ N∗ ;
(ii) 1 + f 2 (n)|n, ∀n ∈ N∗ ;
(iii) f (n) ̸= f (n + 1), ∀n ∈ N∗ .

18.
 Arătaţi că sistemul
 3n + pp = k 2
2p+2 + 33n−1 = 11(p + 2)(3n − 1)
are un unic triplet de soluţii (n, p, k), unde n, p, k ∈ N∗ şi p este un număr prim.

19. Verificaţi dacă există perechi de numere naturale nenule (m, n) astfel ı̂ncât:
n−1
(8m − 1) · n3 + 27m+1 − 7 = 6 · · [1 + 2 + ... + (n + 1)].
n+1


Probleme propuse 67

20. Verificaţi dacă există n ≥ 2 natural şi p prim astfel ı̂ncât 2φ(n)+1 + np = pφ(n) .

pn + 3
21. Demonstraţi că ecuaţia: = n, unde p este un număr prim, iar n un
7
număr natural nenul, are o singură soluţie.

22. Rezolvaţi in N∗ 4 ecuaţia: (k + l)m = k n + lm , dacă k şi l sunt numere prime


şi k ̸= l.

23. Fie ecuaţia (n2 − p) · q + p2 = x2 , unde n şi x sunt numere naturale, iar p şi q
sunt numere prime.
a) Arătaţi că ecuaţia nu are soluţii pentru n ̸≡ x( mod 2).
b) Verificaţi dacă există soluţii ı̂n cazul ı̂n care n şi x au aceeaşi paritate.
c) Determinaţi n şi p ı̂n cazul ı̂n care q = 3 şi x = 2n.

24. Rezolvaţi ı̂n N2 ecuaţia: 2n · n2 = 3m · m3 .

25. Arătaţi că, pentru p ∈ P şi n ∈ N fixate, astfel ı̂ncât p ≥ 11, ecuaţia n2 +pn−1 =
x · px nu are soluţii ı̂n {0, 1, ..., n − 2} .


68 Probleme propuse

X φ(d)
26. Fie funcţia F : N \ {0, 1} → N definită prin: F (n) = . Demonstraţi
d
d|n,d≥2
că F (n) ≤ n − 1 şi precizaţi dacă are caz de egalitate.


 Dacă p, n ∈ N astfel ı̂ncât:
27. hpi
p este pătrat perfect cu +1∈P
q n
2 ,
2n ([ p2 ]+1)
2n + 1 >

6
atunci arătaţi că (p + 2)n + (2n + 1)p+2 nu poate fi pătrat perfect (s-a notat cu [x]
partea ı̂ntreagă a numărului x ).

28. Fie funcţia D : N \ {0, 1} → N, D(n) = gcd(φ(n), τ (n)).


(i) Arătaţi că există o infinitate de numere naturale n ≥ 2 astfel ı̂ncât ecuaţia
D(n) = xk să aibă o soluţie ı̂n N \ {0, 1} , unde k, reprezentând numărul divizo-
rilor primi ai lui x, este fixat.
(ii) Arătaţi că există o infinitate de numere naturale a ≥ 2 astfel ı̂ncât ecuaţia
D(x) = ak să aibă o soluţie ı̂n N \ {0, 1} , unde k, reprezentând numărul divizo-
rilor primi ai lui x, este fixat.

29. Demonstraţi 
că: 
 k X k
τ (n) 1 2  X 1 
≤ τ (d) , ∀n ∈ N, n ≥ 2, d ∈ N (similară cu problema
φ(n) k 3 p −1
i=1 i
d|n
28148 din GM − Bnr.9/2021).


Probleme propuse 69

30. Arătaţi că nu există p, q ∈ P astfel ı̂ncât 11p + 2q şi 10p + q 5 să fie simultan
pătrate perfecte.

31. Găsiţi funcţiile f : N → N astfel ı̂ncât:


i) f (n)f 2 (m) + n · m|(f (n) + n) · f (m + 1) · m, ∀m, n ∈ N∗ ;
ii) f 2 (n) + f 2 (m) + 2mn ≤ f (m + n) · (m + n), ∀m, n ∈ N.

32. Dacă n ∈ N, n ≥ 2 şi acesta are divizorii primi p1 , p2 , ..., pk , atunci demonstraţi
că:
k √ 3
φ(n) Y pi − 1
≥ √ .
τ (n) i=1
3 p
i

33. Determinaţi funcţiile crescătoare f : N∗ → N∗ care ı̂ndeplinesc următoarele


condiţii:
i) f (a)b ≤ f (a)a · f (b), ∀a ∈ N∗ , b ∈ N∗ , 2 | b;
ii) f (a) + f (b) ≤ 2 · f (a)f (b) , ∀a, b ∈ N∗ ;
2
iii) f (a) ≤ aa , ∀a ∈ N∗ , a ∈ P.

34. Determinaţi funcţiile f : N∗ → N∗ care ı̂ndeplinesc următoarele condiţii:



i) f (a)b ≤ a2 f (b) , ∀a, b ∈ N∗ ;
ii) f (a) · b + f (b) · a|(a + b)2 · ab, ∀a, b ∈ N∗ .


70 Probleme propuse

35. Determinaţi funcţiile injective f : P → P astfel ı̂ncât f 2 (p) + 2f (p) · q + f 2 (q)


este
" un pătrat
# perfect, oricare ar fi numerele prime distincte p şi q, iar f (p) ≤
2
(p − 1)
+ 2 pentru orice p prim, unde [x] este partea ı̂ntreagă a unui număr
2
real x.


Capitolul 19

Soluţiile problemelor propuse

1. Din condiţia (i) ⇒ f (f (1)) + f (0)|1, f (f (0)) + f (1)|1 ⇒ f (f (1)) + f (0) =


f (f (0)) + f (1) = 1 ⇒ {f (f (1)), f (0)} = {f (f (0)), f (1)} = {0, 1} .
1
Dacă f (0) = 1 ⇒ 2 · f (1) = 1, f (1) = , f (1) ̸∈ N, ceea ce contrazice ipoteza.
2
Astfel, f (0) = 0 ⇒ f (f (0)) = f (0) = 0 ⇒ f (1) = 1.
Din condiţia (ii) ⇒ f (m) ≤ f (f (m) + f (0)), ∀m ∈ N
⇒ f (m) ≤ f (f (m)), ∀m ∈ N
⇒ f (m) + f (n) ≤ f (f (m)) + f (n), ∀m, n ∈ N
(i)
⇒ f (m) + f (n) ≤ m + n, ∀m, n ∈ N, m + n ̸= 0
(iii)
⇒ f (f (m) + f (n)) ≤ f (m + n), ∀m, n ∈ N, m + n ̸= 0
(ii)
⇒ f (f (m) + f (n)) = f (m + n), ∀m, n ∈ N, m + n ̸= 0
⇒ f (f (m)) = f (m), ∀m ∈ N∗
(i)
⇒ f (m) + f (n)|m + n, ∀m, n ∈ N, m ̸= 0
⇒ f (m) + f (0)|m, ∀m ∈ N∗
⇒ f (m)|m, ∀m ∈ N∗ .

71
72 Soluţiile problemelor propuse

f 2 (1) · a
2. Din prima relaţie, pentru b = 1 ⇒ f 2 (1) · a = f (a) · k, k ∈ N∗ : f (a) = .
k
. f 2 (1) · a ..
Din cea de-a doua relaţie ⇒ pentru b = 1, f (a) .. a · f (1) ⇒ . a · f (1) ⇒
k
k|f (1) : k ∈
{f (1), 1} , f (1) fiind prim.
f (1) · a, a ∈ N∗ \ S
⇒ f (a) =
 f 2 (1) · a, a ∈ S
CazulI : S = ∅
Verificăm: f (a) = f (1) · a; f (a) · b|f 2 (b) · a ⇔ f (1) · ab|f 2 (1) · ab2 , a · f (b)|f (a) · b2 ⇔
ab · f (1)|f (1) · ab2 . Cum ambele relaţii sunt adevărate,
f1 (a) = p · a, ∀a ∈ N∗ , p = f (1)
CazulII : S = N∗
Verificăm: f (a) = f 2 (1) · a; f (a) · b|f 2 (b) · a ⇔ f 2 (1) · ab|f 4 (1) · ab2 , a · f (b)|f (a) · b2 ⇔
ab · f 2 (1)|f 2 (1) · ab2 . Cum ambele relaţii sunt adevărate,
f2 (a) = p2 · a, ∀a ∈ N∗ , p = f (1)
CazulIII : S ̸= N∗ , S ̸= ∅
Atunci există a, b ∈ N∗ astfel ı̂ncât f (a) = f (1) · a, f (b) = f 2 (1) · b. ⇒ f (a) · b|f 2 (b) ·
a ⇔ f (1) · ab|f 4 (1) · ab2 , a · f (b)|f (a) · b2 ⇔ ab · f 2 (1)|f (1) · ab2 ⇔ f (1)|b.
Inversând a cu b ı̂n relaţiile din enunţ ⇒ f (b) · a|f 2 (a) · b ⇔ f 2 (1) · ab|f 2 (1) · ab, b ·
f (a)|f (b) · a2 ⇔ f (1) · ab|f 2 (1) · ba2 , ceea ce este adevărat.
Pentru f (a) = f (1) · a, f (b) = f (1) · b, respectiv pentru f (a) = f 2 (1) · a, f (b) =
f 2 (1)·b, se ştie din cazurile precedente că relaţiile sunt adevărate. Atunci, singura
.
condiţie rămâne că b..f (1), ∀b ∈ S.

 p2 · a, a ∈ S
⇒ f3 (a) = , S ⊆ {np|n ∈ N∗ }
p · a, a ∈ N∗ \ S


Soluţiile problemelor propuse 73

3. Fie P (b, a) : f 2 (b) · f (a)|p2a p2b a2 b2 .P (1, 1) : f 3 (1)|16,


. .
f (1)..2, de unde f 3 (1)..8, f (1) = 2.
P (1, a) : f (a) · 4|4 · p2a a2 ⇒ f (a)|p2a a2 , ∀a ∈ N∗ .
P (a, 1) : f 2 (a)|4 · p2a a2 ⇒ f (a)|2 · pa a, ∀a ∈ N∗ ⇒ f (a)|gcd(pa a, 2) · pa a.
Pentru a prim, a > 2, rezultă că gcd(pa a, 2) = 1, f (a)|pa a, dar cum a şi pa sunt
.
numere prime, iar f (a)..pa ⇒ f (a) ∈ {pa , pa a}
P (b, a( de mai sus )) : oricum ar fi a,
f 2 (b)|pa p2b a2 b2 , f 2 (b)|4p2b b2 , ⇒ f 2 (b)|gcd(pa a2 , 4) · p2b b2 ,
f 2 (b)|p2b b2 , deci f (a)|pa a, ∀a ∈ N∗ .
Pentru f (a) = k·pa , k|a, f (b) = l·pb , l|b : f 2 (b)·f (a)|p2a p2b a2 b2 ⇔ l2 k·p2b pa |p2a p2b a2 b2 ⇔
l2 k|pa a2 b2 , care reiese din faptul că l|b ⇒ l2 |b2 , iar k|a, de unde:
f (a) ∈ {k · pa |k ∈ N∗ , k|a} , ∀a ∈ N∗

4. Dacă y = 0, 5x + 1 = 2z ⇒ (M4 + 1)x + 1 = M4 + 2 = 2z , ⇒ z > 0, z < 2 : z = 1,


de unde: x = y = 0, z = 1 .z ≡ 1 ≡ y + 1( mod 4).
.
Dacă y > 0, 2z ≥ 50 + 3 = 4, z ≥ 2 ⇒ 2z ..4. ⇒ (M4 + 1)x + 3y = M4 , 3y = M4 + 3,
de unde y = 2b + 1, b ∈ N∗ . Dacă x = 0, 9b · 3 = 2z − 1, 2z = (M3 + 2)z = M3 + 1 ⇒
z = 2a, 32b+1 = (2a + 1)(2a − 1), gcd(2a + 1, 2a − 1) = gcd(2a + 1, 2) = 1, deci,
cum 2a + 1 > 2a − 1 şi 32b+1 > 1, 2a + 1 = 32b+1 , 2a − 1 = 1, 2a = 2, 32b+1 = 3 ⇒
x = 0, y = 1, z = 2 .z ≡ 2 ≡ y + 1( mod 4).
.
Dacă x > 0, u(2z − 32b+1 ) = 5.5x , 32b+1 sunt impare, de unde 2z ..2, z ≥ 1.u(2z ) ∈
{2, 4, 6, 8} ,
u(32b+1 ) ∈ {3, 7} , deci u(2z − 32b+1 ) ∈ {5, 9, 7, 1, 3} , fiind egală cu 5 ⇔ u(2z ) = 2
şi u(32b+1 ) = 7, de unde z = M4 + 1, y + 1 = M4 , care nu ı̂ndeplinesc condiţia ca
z ≡ y + 1( mod 4), sau u(2z ) = 8 şi u(32b+1 ) = 3, de unde z = M4 + 3, y + 1 =
M4 + 2, care nu ı̂ndeplinesc nici ele condiţia.


74 Soluţiile problemelor propuse

5. Dacă p − 1 = 1, 13 + 8 = 21 nu este un pătrat perfect. Atunci, p ≡ 1(


.
mod 2), p − 1..2 : p = 2k + 1.
⇒ (13k )2 + p3 = n2 , p3 = (n − 13k )(n + 13k ) = 1 · p3 = p · p2 , astfel ı̂ncât n + 13k >
n − 13k .
Dacă n = 1 + 13k , (2k + 1)3 = 1 + 2 · 13k ⇒ 8k 3 + 12k 2 + 6k = 1 + 2 · 13k + (13k )2 −
(13k )2 = 1 + 2 · 13k , 8k 3 + 3 · 4k 2 + 3 · 2k + 1 = 1 + 2 · 13k ⇒ 4k 3 + 6k 2 + 3k = 13k .
Pentru 13k ≥ 4k 3 + 6k 2 + 3k, 13(k+1) ≥ 13 · (4k 3 + 6k 2 + 3k) = 42k 3 + 78k 2 + 39k >
4(k + 1)3 + 6(k + 1)2 + 3(k + 1) = 4(k 3 + 3k 2 + 3k + 1) + 6(k 2 + 2k + 1) + 3(k + 1) =
4k 3 + 18k 2 + 27k + 13 ⇔ 38k 3 + 60k 2 + 12k > 13, ceea ce este adevărat pentru
k ≥ 1, astfel, cum apare egalitate pentru k = 1 : 13 = 4 + 6 + 3, ecuaţia nu are
soluţii k > 1, iar pentru k = 0 avem p = 1, nu convine, deci singura soluţie este
k = 1 : p = 3, n = 14
Dacă n = p + 13k = p2 − 13k , atunci 2 · 13k = p(p − 1), de unde, cum p ≥
. . .
3, 13k ..p, p ∈ P deci 13..p, p = 13, dar atunci p − 1 = 12..3, dar 2 · 13k ̸≡ 0( mod 3),
contradicţie.

6. pn = np − 3p . Dacă p > 3 şi p ∤ n, np − 3p = np−1 · n − 3p−1 · 3 = Mp + n − 3


.
din mica teorema a lui Fermat. Atunci, n − 3..p, deşi p ∤ n şi p ∤ 3. Astfel se poate
folosi L.T.E: vp (np −3p ) = vp (n−3)+vp (p) = vp (n−3)+1, dar vp (np −3p ) = n, deci
Bernoulli
vp (n − 3) = n − 1, de unde n − 3 ≥ pn−1 = (p − 1 + 1)n−1 ≥ (p − 1)(n − 1) + 1 ≥
n − 1 + 1 > n − 3, contradicţie.
. .
Dacă n..p, atunci 3p = np − pn , 3p ..p. Astfel, p = 3, 33 = n3 − 3n . Fie n = 3k, k ∈
N∗ .33 = 33 · k 3 − 33 · 33k−3 , 1 = k 3 − 33k−3 , deci 33k−3 = k 3 − 1, dar 3k−1 =
Bernoulli
(2 + 1)k−1 2(k − 1) + 1 = 2k − 1 ≥ k, deci k 3 − 1 = 33k−3 ≥ k 3 , contradicţie.

.
Dacă p = 3, 3n + 33 = n3 , astfel n..p şi se ajunge la cazul deja studiat. Dacă
p = 2, 2n + 9 = n2 . Rezultă că n este impar, ı̂nsă atunci 2n + 9 = M3 + (−1)n + 9 =
M3 + 2, dar n2 ≡ 0( mod 3) sau n2 ≡ 1( mod 3), nu convine. Astfel, nu există p
Soluţiile problemelor propuse 75

prim şi n natural nenul care să ı̂ndeplinească relaţia din enunţ.

7. Fie n = pα1 1 pα2 2 ...pαk k descompunerea ı̂n factori primi a lui n.


pn+p p1 p2 ...pk
Se observă că: f (n) = · =
n (p1 − 1)(p2 − 1)...(pk − 1)
pn+p pn+p
= α1 −1 α2 −1 αk −1 ≤ ≤
p1 p2 ...pk (p1 − 1)(p2 − 1)...(pk − 1) (p − 1)k · pα1 +α2 +...+αk −k
k
pn+p k

n+p−k (p − 1 + 1) n+p−k 1
≤ =p · =p · 1+ .
(p − 1)k (p − 1)k p−1
Atunci,
" f (n)f (m) ≤
 k # "  l #
1 1
≤ pn+p−k · 1 + · pm+p−l · 1 + =
p−1 p−1
 k+l  2p  2p
n+p−k+m+p−l 1 n+m−2p+2p 1 n+m 1
=p · 1+ =p · 1+ =p · 1+ .
p−1 p−1 p−1

8. Întrucât m, n ≥ 0 ⇒ 6p ≥ 2, deci p ≥ 1.
Atunci, 3|6p . Cum 7n = (6 + 1)n = M3 + 1, rezultă că 5m = M3 − 1 = M3 + (−1)m ,
de unde există k ∈ N astfel ı̂ncât m = 2k + 1: 5 · 25k + 7n = 6p .
CazulI : p = 1
6 = 5 · 25k + 7k ≥ 5 · 250 + 70 = 5 + 1 = 6. Are loc egalitatea, deci ı̂n acest caz
k=n=0
⇒ m = 1, n = 0, p = 1
CazulII : p ≥ 2
4|6p , deci M4 = 5 · 25k + 7n = M4 + 1 + M4 + (−1)n = M4 + 1 − 1. De aici ⇒ n este
un număr impar, la fel ca m.
Ultima cifră a lui 7n poate fi: u(7M4 +1 ) = u(71 ) = 7 sau u(7M4 +3 ) = u(73 ) =
u(343) = 3.
Ultima cifră a lui 6p este 6, ∀p ≥ 2. Atunci, 5 = u(5·25k ) ∈ {6 − 3, 16 − 7} = {3, 9},
dar acest lucru este fals.
76 Soluţiile problemelor propuse

În acest caz nu există soluţii.

9. Fie gcd(m, n) = d. Atunci există a1 , a2 ∈ N astfel ı̂ncât m = d · a1 si n = d · a2 ,


iar gcd(a1 , a2 ) = 1.
hmi
Presupunem că n > m: atunci = 0 ⇒ m + n|n. Cum n > 0 ⇒ m + n ≤ n, dar
n
m > 0, deci presupunerea este falsă, m ≥ n.
m n
⇒ ≥ , a1 ≥ a2 ; fie a1 = a2 · q + r, unde q, r ∈ N astfel ı̂ncât r ≤ a2 − 1.
d hd i      
m a1 d a1 r
Atunci = = = q+ = q. Din prima relaţie din ipoteză
n a2 d a2 a2
rezultă că:
d(a2 (q + 1) + r)|d(q 2 (a2 q + r) + a2 ) ⇒
o
3 2 3 2
⇒ a2 (q + 1) + r|a2 (q + 1) + r · q şi a2 (q + 1) + r|a2 (q + q ) + r · q 2 ⇒
⇒ a2 (q + 1) + r|a2 (q 2 − 1) ⇒ a2 (q + 1) + r|a2 (q + 1)(q − 1).
Dar cum a1 ≡ r( mod a2 ) şi gcd(a1 , a2 ) = 1 ⇒ gcd(r, a2 ) = 1 ⇒ gcd(a2 (q + 1) +
r, a2 ) = 1. Astfel, a2 (q + 1) + r|(q + 1)(q − 1) (1).
Din a doua relaţie din ipoteză rezultă că: d(a2 (q + 1) + r)|d(q + 1)2 , de unde
a2 (q + 1) + r|(q + 1)2 (2).
Din (1), (2) rezultă, prin scădere: a2 (q + 1) + r|2(q + 1).
CazulI : a2 (q + 1) + r ≤ (q + 1)
⇒ (a2 − 1)(q + 1) ≤ −r ≤ 0 ⇒ a2 ≤ 1, dar n > 0, deci a2 > 0. Atunci a2 = 1 şi
.
0 ≤ −r ≤ 0, de unde r = 0, deci a1 ..a2 , dar gcd(a1 , a2 ) = 1, deci a2 = 1. Rezultă
că n = d, de unde n|m.
CazulII : a2 (q + 1) + r = 2(q + 1)
Ca ı̂n cazul I, a2 > 0. Iar (a2 − 2)(q + 1) ≤ −r ≤ 0, de unde a2 ∈ {1, 2}. Dacă
a2 = 1, atunci r = q + 1 ≥ 1 = a2 , contradicţie. Dacă a2 = 2, atunci r = 0, deci
gcd(a1 , a2 ) = gcd(a2 , r) = 2, nu 1, contradicţie. Acest caz nu convine.


Soluţiile problemelor propuse 77

10. a = 0 ⇔ b0 = 0, care nu este definit (analog daca b = 0 rezultă că a0 = 0, deci


nici b nu poate fi 0). Atunci există x ∈ R, bx = a, astfel ı̂ncât a = bx .
xb x ·bb x+b
Înlocuind ı̂n ecuaţie, obţinem: bx·b·b = bb = bb . Dacă b = 1, atunci şi a = 1.
xb+1 x+b x+b−xb−1 (1−x)(b−1)
Altfel, x · b =b ⇒x=b =b .
Cum b > 1 ⇒ b − 1 > 0, deci (1 − x)(b − 1) scade pe măsură ce x creşte,
funcţia f : R → R, f (x) = b(1−x)(b−1) este strict descrescătoare, iar funcţia g :
R → R, g(x) = x este strict crescătoare. Ecuaţia f (x) = g(x) are atunci cel mult o
soluţie. Se observă că f (1) = 1 = g(1), de unde rezultă că ecuaţia are mulţimea
soluţiilor S = {(a, a)|a ∈ N∗ }.

11. Fie n = pα1 1 pα2 2 ...pαk k descompunerea ı̂n factori primi ai numărului n. Atunci,
1 1 1
φ(n) = n · (1 − )(1 − )...(1 − ) =
p1 p2 pk
α1 α2 αk 1
= p1 p2 ...pk · (p1 − 1)(p2 − 1)...(pk − 1) · =
p1 p2 ...pk
1
= pα1 1 +1 pα2 2 +1 ...pαk k +1 · (p1 − 1)(p2 − 1)...(pk − 1) · 2 2 2 .
p1 p2 ...pk
αi +1
Din inegalitatea lui Bernoulli rezultă că pi ≥ 1 + (pi − 1)(αi + 1) pentru i = 1, k.
Întrucât pi − 1 ≥ 2 − 1 > 0 şi αi + 1 ≥ 1 + 1 > 0, se poate aplica inegalitatea
mediilor:
pαi i +1 ≥ 1 + (pi − 1)(αi + 1) ≥ 2 (pi − 1)(αi + 1), de unde:
p
k k
Y p Y p 1
φ(n) ≥ (2 (pi − 1)(αi + 1)) · (( pi − 1)2 ) · 2 2 2 ,
i=1 i=1
p1 p2 ...pk
k √
Y 2 · ( pi − 1)3 p
 
φ(n) ≥ · (α1 + 1)(α2 + 1)...(αk + 1) =
i=1
p2i
k  √ 
Y pi − 1 3 p
= 2pi · ( ) · τ (n).
i=1
pi
k
" √ 3 #
φ(n) p Y pi − 1
⇒ · τ (n) ≥ 2pi · .
τ (n) i=1
p i


78 Soluţiile problemelor propuse

12. x5 + x7 = 4y · 10 ⇒ x5 (x
2
+ 1) = 4y · 10. Cum:
(M4 )2 + 1 ≡ 1( mod 4)  


2
(M4 + 1) + 1 ≡ 2( mod 4)

⇒ x2 + 1 ̸= M4 , ∀x.
2
(M4 + 2) + 1 ≡ 1( mod 4) 


2

(M4 + 3) + 1 ≡ 2( mod 4)
CazulI : y = 0

x5 (x2 + 1) = 10, deci x5 ≤ 10, x ≤ [ 5 10] = 1. Dacă x = 0 ⇒ 0 = 10, fals. Dacă
x = 1 ⇒ 2 = 10, tot fals. În acest caz nu există soluţii.
CazulII : y ≥ 1
Cum 4 ∤ x2 + 1 si gcd(x5 , x2 + 1) = gcd(x3 , x2 + 1) =
= gcd(x, x2 + 1) = gcd(x, 1) = 1 ⇒ 4|x5 ⇒ x = 2k, k ∈ N. Atunci, k 5 (4k 2 + 1) =
4y−2 · 5.
gcd(4k 2 + 1, 4y−2 ) = gcd(4k 2 + 1, 4) = 1 ⇒ 4k 2 + 1|5, deci 4k 2 + 1 ∈ {1, 5} , 4k 2 ∈
{0, 4} , k ∈ {0, 1} .
(i) k = 0 ⇒ 4y−2 · 5 = 0, dar 4y−2 , 5 > 0, deci acest caz nu are soluţii.
(ii) k = 1 ⇒ 4y−2 · 5 = 5, 4y−2 = 1 = 40 . Atunci y − 2 = 0, iar x = 2 · 1 :
x=y=2

13. Se observă că, dacă m şi n există, atunci


 q m = p2n − (2n + m)2 = (pn − 2n −
 pn − 2n − m = q a
n a m−a
m)(p + 2n + m) = q · q , de unde ⇒
pn + 2n + m = q m−a
⇒ 2 · pn = q a + q m−a , iar a ≤ m − a(a ∈ N).
Dacă q = 2, p ̸= 2, deci vq (2pn ) = 1 = vq (q a + q m−a ) = vq (q a (1 + q m−2a )) =
a + vq (1 + q m−2a ) ⇒ a ≤ 1. Dacă a = 0, pn = 2n + m + 1, deci 4n + 2m + 1 = 2m , de
unde 2 ∤ 2m , m = n = 0, dar m, n ∈ N∗ , nu convine. Dacă a = 1, pn = 1 + 2m−2 =
2m−1 + 2
2n + m + 2 = 2m−2 · 2 − 2n − m; 2n + m = = 2m−2 + 1, 2n =
2
= 2m−2 − m + 1 ≤ 2m ⇒ 2m−2 ≤ 3m − 1. Fie P (m) : 2m−2 > 3m − 1 este adevarată,
pentru m ≥ 3.P (7) : 25 = 32 > 21 − 1 = 20; adevărat. [P (m) → P (m + 1)] :
Soluţiile problemelor propuse 79

2m−2 > 3m − 1 ⇒ 2m−1 > 2(3m − 1) = 6m − 2 > 3m + 6 − 2 = 3m + 4 =


= 3(m + 1) + 1 > 3(m + 1) − 1. Atunci, din inducţie, 2m−2 > 3m − 1, ∀m ≥ 7, astfel
.
ı̂ncât m ≤ 6, m impar (2m−2 −m+1..2) sau m = 2. Dacă m = 5 ⇒ 2n = 8−5+1 =
4, n = 2. Atunci, p2 = 1 + 8 = 9, p = 3, q = 2 . Pentru m = 3, 2n = 2 − 3 + 1 = 0,
nu convine. m = 1 ⇒ 2n = 0.5, n = 0.25, deci n ∈
/ N, nu convine. Dacă
m = 2, 2n = 1 − 2 + 1 = 0, dar n ∈ N∗ .
Dacă q = 3, 2 · pn ̸≡ 0( mod 3) ⇒ a = 0. Dacă p = 2, atunci 2 · pn = 1 + 3m =
M4 , 3m = M4 + (−1)m = M4 − 1, deci m este impar. 2n+1 = 1 + 3m = M3 + 1 =
M3 +(−1)n+1 , deci şi n este impar. u((2n+m)2 ) = u(4n −3m ) = u(4M2 +1 −3M2 +1 ) ∈
{4 − 3, 14 − 7} = {1, 7} , dar, ∀x ∈ N, u(x2 ) ∈ {0, 1, 4, 5, 6, 9} , mulţime din care
7 nu face parte. Astfel, u((2n + m)2 ) = 1, de unde u(3m ) = 3 = u(31 ), deci
m = M4 + 1. Cum 3m = 2 · pn − 1 = 2(2n + m + 1) − 1 = 4n + 2m + 1, 3m =
4·(M2 +1)+2·(M4 +1)+1 = M8 +4+2+1 = M8 +7, dar 3m = 32K+1 = 9K ·3 = M8 +3,
contradicţie. Astfel, p ̸= 2.p ≥ 5 > 3, de unde 2·3n < 2·pn = 1+3m < 2·3m , n < m.
Cum 3m = 4n + 2m + 1, 3m < 6m + 1.
Fie P (m) : 3m ≥ 6m+1 este adevărată. P (3) : 27 > 19, adevărat. [P (m) → P (m + 1)] :
3m ≥ 6m+1 ⇒ 3m+1 ≥ 18m+3 ≥ 6m+13 > 6(m+1)+1, relaţie adevărată. Astfel,
prin inducţie, 3m ≥ 6m + 1, ∀m ≥ 3, deci m ≤ 2. Dacă m = 1, pn = 2 < 5, nu con-
vine. m = 2 ⇒ pn = 5, p = 5 şi n = 1. Verificare: 52 − 32 = (2 + 2)2 ⇔ 25 − 9 = 16,
adevărat. ⇒ p = 5, q = 3 .
Pentru q ≥ 5, analog a = 0, pentru că altfel 2 sau p este divizibil cu q, ceea ce
nu convine. Atunci, pn = 2n + m + 1 şi q m = 4n + 2m + 1, q m + pn = 6n + 3m + 2 =
Bernoulli
(7 − 1) · n + 1 + (4 − 1) · m + 1 ≤ 7n + 4m , deci 5m + pn ≤ q m + pn ≤ 4m + 7n <
5m + 7n , p < 7 ⇒ p ≤ 5.
Bernoulli
q m + 1 ≤ q m + pn = 6n + 3m + 2 ≤ 9m + 2 = (10 − 1)m + 1 + 1 ≤ 10m + 1 ⇒
5 ≤ q ≤ 10, q ∈ {5, 7} .
Dacă q = 5, 5m = 4n + 2m + 1 ≤ 6m + 1. Fie P (m) : 5m > 6m + 1 este adevarată
(m ≥ 1).P (2) : 25 > 13, adevărat. [P (m) → P (m + 1)] : 5m ≥ 6m + 1 ⇒ 5m+1 ≥
80 Soluţiile problemelor propuse

30m + 5 ≥ 6m + 29 > 6(m + 1) + 1, relaţie adevărată. Astfel, prin inducţie,


5m > 6m + 1, ∀m ≥ 2, deci m = 1, 5 = 4n + 3, n = 21 , care nu este un număr
natural.
Dacă q = 7, 7m ≤ 6m + 1, dar pentru P (m) ⇒ 7m > 5m > 6m + 1. Atunci,
m = 1, 7 = 6 + 1 ⇒ m = n = 1, p2 − 7 = (2 + 1)2 = 9 ⇒ p2 = 7 + 9 = 16, p = 4,
deci p nu este prim.

.
14. Cum gcd(m, m + 1) = 1, şi (m + 1)n · (n + 1)..m, m|n + 1.
CazulI : m = n + 1
. . .
Atunci n(n+1) · n! = (n + 2)n ; (n + 2)n ..nn , n + 2..n, de unde 2..n. Dacă n = 1, m =
2 ⇒ 2 = 3 · 2, nu este adevărat. Dacă n = 2, m = 3 ⇒ 8 · 6 = 16 · 3, ceea ce este
adevărat: n = 2, m = 3
n+1
CazulII : m ≤
2
.. n n n n  n  ..
Dacă n.2, atunci m ≤ , dar m ≥ din ipoteză, astfel m = . n 2 · !.n, dar
2 2 2 2
n n (n + 2)n
gcd(( +1)n ·(n+1), n) = gcd(( +1)n , n) = gcd( , n) = gcd((k +1)2k , 2k) =
2 2 2n
n
gcd((k + 1)2k , 2) = gcd(k + 1, 2) ∈ {1, 2} , unde k = . Pentru ca relaţia să fie
2
adevărată, n ∈ {1, 2} . Cum n este un număr par ⇒ n = 2, m = 1, ⇔ 21 ·1 = 22 ·3,
care nu este adevărat.
n+1 n−1 hni
Dacă 2 ∤ n, m = sau m ≤ = ≤ m, deci apare egalitatea. Dacă
2 2 2
n+1
m= , atunci n = 2m−1, de unde (2m−1)m ·m! = (m+1)2m−1 ·2m, (2m−1)m ·
2
(m−1)! = (m+1)2m−1 ·2. Cum gcd(m−1, m+1)|2, cum gcd(m−1, m+1)|2, dacă nu
există k ∈ N astfel ı̂ncât m−1 = 2k , atunci gcd(m−1, (m+1)2m−1 ·2) ̸= m−1, m−1 ∤
(m+1)2m−1 ·2, deci nu există soluţii ı̂n acest caz. Dacă m = 2k +1, k ∈ N, relaţia se
k +1 k+1 +1 k +1
reduce la: (2k+1 +1)2 ·(2k +1)! = (2k +2)2 ·2·(2k +1) ⇒ (2k+1 +1)2 ·(2k )! =
k+1 +1 k+1 +2
(2k−1 +1)2 ·22 . Cum gcd(2k+1 +1, 2l ) = 1, ∀l ∈ N, gcd(2k+1 +1, 2k−1 +1) =
k +1
gcd(2k+1 + 1, 2k+1 + 4) = gcd(2k+1 + 1, 4) = 1, de unde gcd((2k+1 + 1)2 , (2k−1 +
Soluţiile problemelor propuse 81

k +1 k+1 +2 k +1 k +1 k +2
1)2 · 22 ) = 1. Din relaţie ı̂nsă, (2k+1 + 1)2 |(2k−1 + 1)2 · 22 , de unde
2k +1 2k+1 +1
(2k+1 + 1) |1, (2k+1 + 1) = 1, 2k sau 2k+1 = 0, care nu are soluţii naturale.
n−1
Dacă m = , n = 2m + 1, (2m + 1)m · m! = 2 · (m + 1)2m+2 , ı̂nsă gcd(2m + 1, m +
2
1) = gcd(m, m+1) = 1, iar gcd(2m+1, 2) = 1, de unde gcd(2m+1, 2·(m+1)2m+2 ) =
1, dar cum 2m+1|2·(m+1)2m+2 , 2m+1 = 1, m = 0, care nu este un număr natural
nenul, deci nu poate fi o soluţie.

15. Fie descompunerea ı̂n factori primi a numărului n, n = pα1 1 pα2 2 ...pαk k : nx =
xαk
pxα 1 xα2 x
1 p2 ...pk , τ (n) = (α1 + 1)...(αk + 1), τ (n ) = (xα1 + 1)...(xαk + 1).
 k
x x · (α1 + α2 + ... + αk ) + k
Din inegalitatea mediilor, τ (n ) ≤ =
k
 k
α1 + α2 + ... + αk Bernoulli α1 +α2 +...+αk
·k
= x· +1 ≤ (x + 1) k =
k
k k 1 2 k 1 k
= (x + 1)α1 +α2 +...+αk +2 −k−2 +k = (x + 1)α1 +α2 +...+αk +(Ck +Ck +...+Ck )·1−Ck −2 +k =
2 k k +k
= (x + 1)α1 +α2 +...+αk +(Ck +...+Ck )·1−2 ≤
1+ ki=1 αi + 1≤i<j≤k αi αj +...+α1 α2 ...αk k +k k +k−1
P P
≤ (x+1) ··x−1−2 = (x+1)(α1 +1)...(αk +1)−2 =
τ (n)−1−2k +k τ (n) k
= (x + 1) ≤ (x + 1) ⇔ 2 ≥ k − 1,
∀k ∈ N ⇔ Ckk + Ckk−1 + ... + Ck1 + 1 ≥ k − 1 ⇔ Ckk + Ckk−1 + ... + Ck2 + 2 ≥ 0. Dacă
k ≥ 2, relaţia este adevărată. În caz contrar, k − 1 ≤ 0 < 2k , deci relaţia este tot
adevărată.
Egalitatea are loc atunci când 2k = k − 1, dar se observă din relaţia anterioară că
acest lucru nu este posibil.

√ .
16. Fie [ n] = k. Atunci, k 3 ..n, unde k 2 < n < (k + 1)2 = k 2 + 2k + 1, deci k 2 <
k3 k3 k3
n ≤ k 2 + 2k.n = , d ∈ N∗ , astfel ı̂ncât = k2 < k2 + 1 ≤ n = ≤ k(k + 2) =
d k d
k · (k 2 − 4) k 3 − 4k k3 k3 k3 k3 1 1
= < , pentru k > 2. Astfel, < < , < <
k−2 k−2 k−2 k d k−2 k d
82 Soluţiile problemelor propuse

1
⇒ k > d > k − 2, d ∈ N∗ , astfel d = k − 1.gcd(k 3 , k − 1) = gcd(k, k − 1) = 1,
k−2
iar din k > 2 rezultă k − 1 ≥ 2, k 3 nu poate fi divizibil cu k − 1, deci pentru k ≥ 3
nu există soluţii.
În continuare avem un set de numere finit n ∈ {0, 1, 2, 3, 4, 5, 6, 7, 8 = 32 − 1} ,
n
pentru fiecare existând maxim o soluţie pentru ecuaţia 2m = √ 3 , relaţia iniţială
[ n]
generând un număr finit de perechi de soluţii (m, n).

17. Cum k ∈ P, din cea de-a treia proprietate obţinem că k este perioada princi-
pală: f (n) = f (n + k), ∀n ∈ N∗ .



 1 + f (n)|n2 , 1 + f (n)|2 · n2
Astfel, 1 + f (n)|2 · (n + k)2 (= 2n2 + 4nk + 2k 2 )


1 + f (n)|(n + 2k)2 (= n2 + 4nk + 4k 2 )


1 + f (n)|4nk + 2k 2
⇒ ⇒ 1 + f (n)|2k 2 , ∀n (1)
1 + f (n)|4nk + 4k 2

 1 + (f (n))2 |n
şi ⇒ 1 + (f (n))2 |k, ∀n (2)
1 + (f (n))2 |n + k
Cum k ∈ P ⇒ 1 + (f (n))2 ∈ {1, k} . Dacă k − 1 = x2 nu are soluţii ı̂n Z, atunci
f (n) = 0 = f (n + 1), care contrazice ipoteza, astfel existând x ∈ N pentru care
x2 = k − 1.
Pentru
 f (n) = x
1 + x|2k 2 = 2 · (x2 + 1)2 = 2x4 + 4x2 + 2

 1 + x|(1 + x) · 2x3 = 2x4 + 2x3

 1 + x|2x3 − 4x2 − 2

1 + x|(1 + x) · 2x2 = 2x3 + 2x2

 1 + x|6x2 + 2

1 + x|(1 + x) · 6x = 6x2 + 6x
Soluţiile problemelor propuse 83

 1 + x|6x − 2
⇒ ⇒ 1 + x|8,
1 + x|6(1 + x) = 6x + 6
1 + x ∈ {1, 2, 4, 8} ⇒ x ∈ {0, 1, 3, 7} , x2 ∈ {0, 1, 9, 49} , k ∈ {1, 2, 10, 50} ∩ P = {2} .
Astfel, k = 2 ⇒ x = 1, f (n) ∈ {0, 1} , iar cum f (n) ̸= f (n + 1), ∀n ∈ N∗ ,
pentru f (n) = 0 ⇒ f (n + 2k) = 0, ∀k ∈ N, iar f (n + 2k + 1) = 1, ∀k ∈ N. Cum

0, 2 ∤ n
1 + f (1)|1, f (1) = 0. Pentru că 2 ∤ 1 : f (n) =
 1, 2|n

.
18. Presupunem că n este impar. Atunci, 3n − 1 este par, 2p+2 + 33n−1 ..2, dar
fiind suma dintre un număr par şi unul impar este un număr impar. Astfel, pre-
supunerea este falsă, n este par.
Fie n = 2a, a ∈ N∗ . Atunci, pp = k 2 − 32a = (k − 3a )(k + 3a ). Dacă p ∤ gcd(k −
3a , k + 3a ), cum p este prim, k − 3a = 1 şi k + 3a = pp , de unde k = 1 + 3a , 2 · 3a =
pp − 1 = pp − 1p , deci 3 ∤ p şi 2 ∤ p. Din L.T.E., v3 (pp − 1p ) = v3 (p − 1) = a, de unde
. . .
p − 1..3a , p − 1 este par, deci p − 1..2 · 3a , p − 1..pp − 1, dar p > 1, fiind prim, deci
p − 1 ≥ pp − 1, p ≥ pp , 1 ≥ p, contradicţie. În acest caz nu există soluţii.
Dacă p|k − 3a si p|k + 3a , atunci p|gcd(k − 3a , k + 3a ) = gcd(k + 3a , 2 · 3a ) ⇒ p|2 · 3a .
.
Cum p este prim, p ∈ {2, 3} . Dacă p = 2, p + 2..2, dar 2p+2 + 33n−1 este impar,
. .
2p+2 + 33n−1 ..p + 2, deci nu convine. Dacă p = 3, 32a + 27 = k 2 , de unde k ..3.
.
Fie k = 3l. Atunci, 9a + 27 = 9 · l2 , 9a−1 + 3 = l2 . Dacă a ≥ 2, 9a−1 ..9, deci
l2 = M9 + 3, l2 este un pătrat perfect divizibil cu 3, dar nu cu 9, nu convine.
Astfel, a ≤ 1, a = 1.n = 2, k 2 = 9 + 27 = 36, deci k = 6. Se verifică cea de-a doua
relaţie din enunţ: 25 + 35 = 11 · 5 · 5 ⇔ 32 + 243 = 11 · 25 ⇔ 275 = 275, adevărat.
Sistemul are atunci un unic triplet de soluţii: n = 2, p = 3, k = 6 .


84 Soluţiile problemelor propuse

n−1
19. Dacă (8m − 1) · n3 + 27m+1 − 7 = 6 · · [1 + 2 + ... + (n + 1)] ⇒ (8m − 1) ·
n+1
3 m+1 n − 1 (n + 1)(n + 2)
n + 27 −7 = 6· · = 3 · (n − 1)(n + 2) = 3n2 + 3n − 6 ⇒
n+1 2
8m · n3 + 27m+1 = n3 + 3n2 + 3n − 6 + 7 = n3 + 3n2 + 3n + 1 = (n + 1)3 .
Atunci, (2m · n)3 + (3m+1 )3 = (n + 1)3 . Cum m, n ∈ N, 2m · n, 3m+1 , n + 1 ∈ N∗ . Din
Marea Teorema a lui Fermat ı̂nsă, ecuaţia aceasta nu are soluţii, ı̂ntrucât 3 > 2,
deci nu există perechi (m, n) de numere naturale care ı̂ndeplinesc condiţia dată.

. .
20. Dacă există q ∈ P astfel ı̂ncât q ̸= p, q ̸= 2 şi n..q, atunci φ(n)..(q − 1). Din mica
φ(n) φ(n)
teorema a lui Fermat rezultă că (2 q−1 )q−1 · 2 + np = Mq + 2 = (p q−1 )q−1 = Mq + 1,
. .
de unde q | 1, dar este număr prim, deci nu convine. Dacă n..2 ⇒ p..2, p = 2, dar
. . .
2φ(n)+1 > 2φ(n) , astfel n < 0, nu convine. Dacă n..p, 2φ(n)+1 ..p, p = 2, deci n..2, nu
convine. Astfel, ∀p ∈ P, n ̸≡ 0( mod p) ⇒ n = 1, dar n ≥ 2, contradicţie. n şi p
nu există.

21. Dacă p ≥ 7, pn + 3 ≥ 7n + 3 > 7n ≥ 7n ⇔


⇔ 7n−1 ≥ n.71−1 ≥ 1, iar 7n−1 ≥ n implică 7n ≥ 7 · n > n + 1 dacă n ≥ 1. Astfel,
prin inducţie rezultă că 7n−1 ≥ n, ∀n ∈ N∗ , deci ı̂n aceste condiţii nu există soluţii
naturale nenule. Se observă că p ≤ 5.
CazulI : p = 5
Fie funcţia w : N → N∗ , w(n) = 5n + 3 − 7n, ∀n ∈ N∗ . Arătăm că w(n + 1) >
w(n), ∀n ∈ N∗ :
5n+1 + 3 − 7(n + 1) > 5n + 3 − 7n ⇔ 5n · 4 > 7. Cum n ≥ 1, 5n · 4 ≥ 51 · 4 = 20 > 7,
de unde rezultă concluzia.
Atunci, w(n) ≥ w(1) = 5 + 3 − 7 = 1 > 0. În acest caz nu există soluţii.
CazulII : p = 3
Soluţiile problemelor propuse 85

Procedând analog ca ı̂n cazul precedent, redefinim w : N → N∗ , w(n) = 3n + 3 −


7n, ∀n ∈ N∗ . De aceasta data ı̂nsă arătăm că w(n + 1) > w(n) + 2, ∀n ∈ N, n ≥ 2 :
3n+1 + 3 − 7(n + 1) > 3n + 5 − 7n ⇔ 3n · 2 > 9. Dacă n ≥ 2, 3n · 2 ≥ 32 · 2 = 18 > 9,
de unde rezultă concluzia.
Atunci, dacă n ≥ 3, w(n) > 2 + w(2) = 11 + 3 − 14 = 0 > 0, w(2) = 9 + 3 − 14 =
−2 < 0, iar w(1) = 3 + 3 − 7 = −1 ̸= 0. Astfel, nici ı̂n acest caz nu există soluţii.
CazulIII : p = 2
Analog, redefinim w : N → N∗ , w(n) = 2n + 3 − 7n, ∀n ∈ N∗ . În acest caz arătăm
că w(n + 1) > w(n), ∀n ∈ N, n ≥ 3 :
2n+1 + 3 − 7(n + 1) > 2n + 3 − 7n ⇔ 2n > 7. Cum n ≥ 3, 2n ≥ 23 = 8 > 7, de unde
rezultă concluzia.
Atunci, w(n) ≥ w(6) = 64 + 3 − 42 = 25 > 0, ∀n ∈ N, n ≥ 6, iar w(1) = 2 + 3 − 7 =
−2 ̸= 0, w(2) = 4 + 3 − 14 = −7 ̸= 0, w(3) = 8 + 3 − 21 = −10 ̸= 0, w(4) =
16 + 3 − 28 = −9 ̸= 0, dar w(5) = 32 + 3 − 35 = 0. Astfel, ecuaţia are o unică
pereche de soluţii (p, n) ∈ P × N : p = 2, n = 5

22. Din L.T.E. (Lifting the Exponent Lemma), cum k ∈ P şi (k + l)m − lm =
k n , gcd(k + l, k) = gcd(l, k) = 1, vk ((k + l)m − lm ) = vk (k) + vk (m) = n, astfel m =
k n−1 · q, unde gcd(k, q) = 1.(k + l)m ≥ k m + lm dacă m ≥ 1, din cauza termenilor
de grade mai mici. Atunci, (k + l)m − lm = k n ≥ k m , n ≥ m = k n−1 · q ≥ 2n−1 .
Dacă n = 1, 20 = 1, n = 2 ⇒ 21 = 2 Dacă 2n−1 ≥ n, 2n ≥ 2n > n + 1 pentru
n ≥ 2. Atunci, dacă n ≥ 3, n < 2n−1 , deci nu există soluţii. Dacă n = 1 sau
n = 2, apare cazul de egalitate: q = 1, k = 2, dacă n = m = 2 atunci 22 + l2 =
(l + 2)2 , 22 + 2 · 2 · l = 4, l = 0 ∈
/ P. Dacă n = m = 1, l şi m pot lua orice valoare ı̂n
P : m = n = 1, k, l ∈ P, k ̸= l


86 Soluţiile problemelor propuse

23. a) (n2 − p) · q + p2 = x2 ⇔ (n2 − p) · q = (x − p)(x + p).


Dacă x−p = q·a, n2 −p = a·(qa+2p) ⇒ a2 ·q+2ap−(n2 −p) = 0, n2 = q·a2 +2ap+p şi
n 2 − a2 · q
p= . Dacă n ≡ a ≡ 0( mod 2), atunci p ≡ 0( mod 2), deci x ≡ 0 ≡ n(
2a + 1
mod 2), care contrazice ipoteza. Dacă n ≡ 0( mod 2), a ≡ 1( mod 2), atunci
fie q = 2, p ≡ 0( mod 2), de unde x ≡ 0 ≡ n( mod 2), deci nu convine, fie
q ≡ 1( mod 2), de unde p ≡ 1( mod 2), deci x ≡ 0 ≡ n( mod 2), care din nou
nu convine. Dacă n ≡ 1( mod 2), a ≡ 0( mod 2), atunci p ≡ 1( mod 2), deci
x ≡ 1 ≡ n( mod 2), iar, dacă n ≡ a ≡ 1( mod 2), fie q ≡ 1( mod 2), de unde
p = 2 şi x ≡ 1 ≡ n( mod 2), fie q ≡ 0( mod 2), de unde p ≡ 1( mod 2) şi deci
x ≡ 1 ≡ n( mod 2), care contrazice ipoteza. Astfel, ı̂n acest caz nu există soluţii.
Dacă x+p = q·a, n2 −p = a·(qa−2p) ⇒ a2 ·q−2ap−(n2 −p) = 0, n2 = q·a2 −2ap+p şi
a2 · q − n 2
p= . Dacă n ≡ a ≡ 0( mod 2), atunci p ≡ 0( mod 2), deci x ≡ 0 ≡ n(
2a − 1
mod 2), care nu ı̂ndeplinesc ipoteza. Dacă n ≡ 0( mod 2), a ≡ 1( mod 2), atunci
fie q = 2, p ≡ 0( mod 2), deci x ≡ 0 ≡ n( mod 2), care nu convine, fie q ≡ 1(
mod 2), de unde p ≡ 1( mod 2), deci x ≡ 0 ≡ n( mod 2), care iar nu convine.
Dacă n ≡ 1( mod 2), a ≡ 0( mod 2), atunci p ≡ 1( mod 2), deci x ≡ 1 ≡ n(
mod 2), dacă n ≡ a ≡ 1( mod 2), fie q ≡ 1( mod 2), de unde p = 2 şi x ≡ 1 ≡ n(
mod 2), fie q ≡ 0( mod 2), de unde p ≡ 1( mod 2) şi deci x ≡ 1 ≡ n( mod 2),
care contrazice ipoteza. Nici ı̂n acest caz nu există soluţii.
b) Dacă p = q = 5 şi n = x = 0, atunci relaţia se verifică.
c) Dacă q = 3, x = 2n, atunci 3 · n2 − 3p + p2 = 4 · n2 , p2 − 3p − n2 = 0, de
unde ∆ = 9 + 4 · n2 = 9 + (2n)2 = s2 , ı̂ntrucât p ∈ Z rezultă că s ∈ N. Atunci,
(s − 2n)(s + 2n) = 9 = 1 · 9 = 3 · 3. Dacă s − 2n = 1, s + 2n = 9, s = 5 şi atunci
3±5
p= ∈ {−1, 4} , dar −1, 4 ∈
/ P, contradicţie. Dacă s − 2n = s + 2n = 3, s = 3
2
3±3
şi p = ∈ {0, 3} , dar cum p ∈ P rezultă că p = 3, n = 0
2

. .
24. m = 0 ⇔ n = 0. Dacă m, n > 0, atunci 3m · m3 ..2, deci m..2, m = 2k,
Soluţiile problemelor propuse 87

. . .
analog 2n · n2 ..3, n..3, n = 3l.gcd(2, 3) = 1, deci m3 ..23l , m = 2k = 2l · s, analog
.. 2k
n2 .3 , n = 3l = 3k · t.
Atunci, 2n ·3m ·t2 = 3m ·2n ·s2 , de unde s2 = t2 , s, t > 0, deci s = t : k = 2l−1 ·s, l =
k−1 ·s−1
3k−1 · s, de unde k = 23 · s. În continuare, arătăm că 3k−1 ≥ k, ∀k ∈ N∗ :
Pentru k = 1, 30 = 1. Dacă 3k−1 ≥ k, atunci 3k ≥ 3k > k + 1, astfel prin inducţie
3k−1 ≥ k, ∀k ∈ N∗ , cu egalitate ⇔ k = 1. (1)
Analog, arătăm că 2k−1 ≥ k, ∀k ∈ N∗ : pentru k = 1, 20 = 1, pentru k = 2, 21 = 2.
Dacă 2k−1 ≥ k, atunci 2k ≥ 2k > k + 1 dacă k ≥ 2, astfel prin inducţie 2k−1 ≥
k, ∀k ∈ N∗ , cu egalitate ⇔ k = 1 sau k = 2. (2)
Din (1) şi (2), k ≥ 2ks−1 · s ≥ 2k−1 ≥ k, cu egalitate ⇔ k = s = 1. Astfel, l = 30 =
1, n = 3 şi m = 2, perechile de soluţii ı̂n N2 fiind: (n, m) ∈ {(0, 0), (3, 2)}

25. Presupunând că are, fie d = n − 1 − x, astfel d ≥ 1.(x + d + 1)2 + px pd =


px · x, (x + d + 1)2 = px (x − pd ), deci x > pd . Notăm D = x − pd ≥ 1.
d +D
Atunci, (pd + D + d + 1)2 = D · pp . Cum d ≥ 1, p ≥ 2, pd > 2d , astfel 2d ≥
d + 1, ∀d ∈ N∗ , din inducţie: pentru d = 1, 2 ≥ 2; iar dacă 2d ≥ d + 1, atunci
2d+1 ≥ 2(d + 1) = 2d + 2 ≥ d + 2.
2 r !2
4 · pd + 2D
 ma ≤mp
pd +D d
2 16 · p2d + 4D2
p ·D ≤ 2·p +D = ≤ = 8 · p2d +
2 2
2
2D .
De asemenea, pd > 2d ≥ 2d ⇔ 2d−1 ≥ (d − 1) + 1, care am demonstrat deja că este
adevărat pentru d ≥ 2, iar pentru d = 1 avem: 20 = 1 ≥ 1.
d +D
Astfel, pd > 2d, de unde 8 · p2d + 2D2 ≥ pp · D > p2d+D · D.
Rezultă că 2D2 > p2d · (pD · D − 8), dar p2d ≥ 112 = 121, şi cum D > 0 atunci
pD ≥ D + 1, astfel 2D2 > 121 · (D2 + D − 8) = 121 · D2 + 121(D − 8), deci
121(8 − D) > 119 · D2 .
Dacă D ≥ 8, atunci relaţia este falsă, deci ecuaţia nu are soluţii.
88 Soluţiile problemelor propuse

Altfel, 2D2 ≤ 2 · 49 = 98, iar p2d · (pD · D − 8) ≥ 121 · (p − 8) ≥ 121 · (11 − 8) =


121 · 3 = 363 > 98 ≥ 2D2 , deci presupunerea este falsă şi nu pot exista soluţii ı̂n
mulţimea dată.

26. Fie n = pα1 1 pα2 2 ...pαk k descompunerea  ı̂n factori primi a numărului n. Atunci,
 
k X φ(d)
X X  X φ(d)  X φ(d)
F (n) = +  +...+ =
  
d d d

α
.. .
i=1 i 1≤i<j≤k
 
d|pi ,d≥2
d|n,d..p1 p2 ...pk
α α
d|pi i pj j ,d.pi pj
     
1 1 1
= α1 · 1 − + α2 · 1 − + ... + αk · 1 − +
p1   p2  pk   
X 1 1 1 1
+ αi αj 1 − 1− + ... + α1 α2 ...αk 1 − ... 1 − +1−1 =
1≤i<j≤k
p i p j p 1 p k
       
1 1
= α1 1 − + 1 ... αk 1 − +1 −1=
p1 pk
1 1
= ·[α1 (p1 − 1) + p1 ] ... [αk (pk − 1) + pk ]−1 = ·[(α1 + 1)(p1 − 1) + 1]·
p1 p2 ...pk p1 p2 ...pk
Bernoulli 1
... · [(αk + 1)(pk − 1) + 1] − 1 ≤ · pα1 +1 ...pαk k +1 − 1 = pα1 1 pα2 2 ...pαk k − 1 =
p1 p2 ...pk 1
= n − 1.
Egalitatea apare atunci când p1 = p2 = ... = pk = 1 ∈
/ P, deci nu poate avea loc.

27. CazulI : 2 ∤ p
⇒ p = M4 + 1, fiind un pătrat perfect.
(i) 2|n ⇒ (p + 2)n + (2n + 1)p+2 = (M4 + 3)M2 + M4 + 1 = M4 + 2, deci nu poate fi
un pătrat perfect.
(ii) 2 ∤ n ⇒ (p + 2)n + (2n + 1)p+2 = (M4 + 3)M2 +1 + (M4 + 3)M2 +1 = M4 + 2, deci
nu poate fi un pătrat perfect.
Soluţiile problemelor propuse 89

CazulII : 2|p
hpi
⇒ p = 4y 2 , 2y 2 = . Atunci, cum 2y 2 + 1 este un număr prim, ı̂l notăm cu
2
q ∈ P.
r r r
2n · q n 2n · 2n 4n
Cum 2n + 1 > ≥ = ⇒ 6 · (2n + 1)2 > 4n , 6 · (4n2 + 4n + 1) >
6 6 6
4n ⇒ 24 · n2 + 24 · n + 6 > 4n .
Fie P (n) : 4n ≥ 24 · n2 + 24 · n + 6 este adevărată, n ∈ N∗ . P (6) : 46 = 4096 >
24 · 36 + 24 · 6 + 6 = 1014, adevărat. [P (n) → P (n + 1)] : 4n ≥ 24 · n2 + 24 · n + 6 ⇒
4n+1 ≥ (24 · n2 + 24 · n + 6) · 4 ≥ 24 · (n + 1)2 + 24 · (n + 1) + 6 ⇔ 96 · n2 + 96 · n + 24 ≥
24 · n2 + 48 · n + 24 + 24 · n + 30 ⇔ 72 · n2 + 24 · n ≥ 30, relaţie adevărată când
n ≥ 1.
Astfel, prin inducţie, 4n ≥ 24 · n2 + 24 · n + 6, ∀n ≥ 6, astfel ı̂ncât n ≤ 5 (1).
.
Dacă 3 ∤ y, y 2 ≡ 1( mod 3), 2y 2 ≡ 2( mod 3), q ..3, q prim, deci q = 3, y = 1, p =
4.(p + 2)n + (2n + 1)p+2 = 6n + (2n + 1)6 .n ≤ 5. Pentru n = 5, u(6n + (2n + 1)6 ) =
u(6 + 1) = 7,rcare nu poate
r fi ultima cifră a unui pătrat perfect. Pentru n =
n
2 ·q n 24 · 34 √ 3 √
4, 2n+1 = 9, = = 6 = 216 > 9 = 2n+1, nu convine. Pentru
6 6
n = 3, 6 + (2n + 1) = 6 + 7 = 216 + 493 = M25 + 16 − 1 = M25 + 15, 6n + (2n + 1)6
n 6 3 6

este divizibil cu 5, care este prim, dar nu şi cu 25 = 52 , deci nu poate fi un pătrat
perfect. Dacă n = 2, 6n + (2n + 1)6 = 62 + 56 = 15661, care nu este pătrat perfect.
n = 1 ⇒ 6n + (2n + 1)6 = 6 + 36 = M9 + 6, fiind divizibil cu 3, care este prim, dar
nu şi cu 9 = 32 , acesta nu poate fi pătrat perfect.
Dacă
r 3|y, cum q ≥ 2 > 1 ⇒ y ̸= 0, y ≥ 3, q ≥ 2 · 9 + 1 = 19. Atunci, 2n + 1 >
2n · 19n
⇒ 6 · (4n2 + 4n + 1) > 38n ⇒ 24 · n2 + 24 · n + 6 > 38n .
6
Din relaţia (1) avem că n ≤ 5. Dacă n = 5, 24 · n2 + 24 · n + 6 = 24 · 25 + 24 · 5 + 6 =
726 < 900 < 382 < 38n , nu convine. Dacă n = 4, 24·n2 +24·n+6 = 24·16+24·4+6 =
486 < 382 < 38n , contradicţie. n = 3 ⇒ 24 · n2 + 24 · n + 6 = 24 · 9 + 24 · 3 + 6 =
294 < 382 < 38n , din nou apare contradicţia. Dacă n = 2 ⇒ 24 · n2 + 24 · n + 6 =
24 · 4 + 4 · 2 + 6 = 150 < 382 = 38n , nu convine. Atunci, n ≤ 1, iar cum
90 Soluţiile problemelor propuse

n ∈ N∗ ⇒ n = 1 ⇒ (p + 2)n + (2n + 1)p+2 = p + 2 + 3p+2 = M3 + 2, deci nu poate


fi pătrat perfect.

28. (i) Considerând n = p1 p2 ...pk un număr impar, liber de pătrate, rezultă că
φ(n) = (p1 −1)(p2 −1)...(pk −1), unde fiecare dintre cele k paranteze este divizibilă
.
cu 2 ( p1 , p2 , ..., pk fiind numere impare), de unde φ(n)..2k , iar τ (n) = (1 + 1)k = 2k ,
deci D(n) = 2k , x = 2. Cum există o infinitate de numere prime, se vor lua câte
k numere prime consecutive pentru a forma un n, apoi următoarele k şi aşa mai
departe, de unde concluzia.
(ii) Considerând că a este un număr prim, ı̂l vom nota cu p, căruia ı̂i vom asocia
(p−1) (p−1) (p−1)
numărul n = p1 p2 ...pk , cu p1 = p. Astfel, φ(n) = p(p−2) · (p2 p3 ...pk )(p−2) ·
(p−1)(p2 −1)(p3 −1)...(pk −1), iar τ (n) = pk . Luând p−2 ≥ k, rezultă că D(n) = pk ,
de unde concluzia (k fiind finit şi mulţimea numerelor prime fiind nemărginită la
dreapta, ı̂nseamnă că există o infinitate de numere prime mai mari ca k + 2).

29. Fie n = pα1 1 pα2 2 ...pαk k descompunerea numărului n ı̂n factori primi, unde α1 ,
α2 ,... , αk ∈ N∗ . Atunci, ∀d ∈ N astfel ı̂ncât d|n, d = pβ1 1 pβ2 2 ...pβkk , unde βi ∈
{0, 1, ..., αi }, ∀i ∈ {1, 2, ..., k}.
Astfel, τ (d) = (β1 + 1)(β2 + 1)...(βk + 1).
X (α1 + 1)(α1 + 2)
Dacă k = 1: τ (d) = (0 + 1) + (1 + 1) + ... + (α1 + 1) = . (1)
2
d|n
X X X X
Dar τ (d) = τ (d) · τ (d), iar τ (d) = τ (1) + τ (pk ) + ... +
α α α α α
d|n k−1
d|p1 1 p2 2 ...pk−1 d|pk k d|pk k
(αk + 1)(αk + 2)
τ (pαk k ) = 1 + 2 + ... + (αk + 1) = , deci
2
X (αk + 1)(αk + 2) X
τ (d) = · τ (d), ∀k ∈ N∗ .
α1 α2 αk 2 α1 α2 αk−1
d|p1 p2 ...pk d|p1 p2 ...pk−1
Soluţiile problemelor propuse 91

k
X Y (αi + 1)(αi + 2)
Astfel, dacă τ (d) = , atunci
α α α i=1
2
d|p1 1 p2 2 ...pk k

k+1
X Y (αi + 1)(αi + 2)
τ (d) = (2)
α α αk+1 i=1
2
d|p1 1 p2 2 ...pk+1

k
(1),(2) X Y (αi + 1)(αi + 2)
⇒ τ (d) = , ∀k ∈ N∗ .
α α α i=1
2
d|p1 1 p2 2 ...pk k
k
X Y (αi + 1)(αi + 2)
Rezultă că τ (d) = =
i=1
2
d|n
 k k  k k
1 Y 1 Y
= (α1 + 1)(α2 + 1)...(αk + 1) (αi + 2) = τ (n) (αi + 2).
2 i=1
2 i=1
 
 k X k
1 2  X 1 
Atunci, τ (d) =
k 3 i=1 i
p −1
d|n
" k  #
Y 2 αi + 2  1 mh≤mg
= τ (n) · · · k

i=1
3 2 1 1 1
+ p −1 +...+ p −1
p −1
" k   # 1 2 i

mh≤mg Y αi + 2 1 αi ≥1
≥ τ (n) · ·p ≥
i=1
3 k
(p1 − 1)(p2 − 1)...(pk − 1)
" k  #
αi ≥1 Y 1 + 2 1
≥ τ (n) · ·p =
i=1
3 k
(p1 − 1)(p2 − 1)...(pk − 1)
1
= τ (n) · 1k · p ≥
1 − 1)(p2 − 1)...(pk − 1)
k
" k  (p
#
Y 1 1
≥ τ (n) · αi −1
p ≥
i=1
pi k
(p1 − 1)(p2 − 1)...(pk − 1)
1 1
≥ τ (n) · α1 −1 α2 −1 αk −1 · =
p1 p2 ...pk (p1 − 1)(p2 − 1)...(pk − 1)
1 1
= τ (n) · α α αk (p1 −1)(p2 −1)...(pk −1)
= τ (n) · k =
(p1 p2 ...pk ) ·
1 2 
p1 p2 ...pk
Y p i − 1

i=1
pi
92 Soluţiile problemelor propuse
 
 k X k
1 τ (n) τ (n) 1 2  X 1 
= τ (n) · = .⇒ ≤ · τ (d) .
Yk 
1
 φ(n) φ(n) k 3 p −1
i=1 i
d|n
n· 1−
i=1
pi

30. Presupunem că este posibil ca cele două expresii să fie simultan pătrate
perfecte. Astfel, din prima relaţie rezultă că, pentru 11p + 2q = k 2 , u(k 2 ) ∈
{0, 1, 4, 5, 6, 9} ⇒ u(2q ) ∈ {9, 0, 3, 4, 5, 8} ∩ {2, 4, 8, 6} = {4, 8} ⇒ q ≡ 2( mod 4)
sau q ≡ 3( mod 4).(1)
.
În cea de-a doua relaţie, p ≥ 2, deci 10p ..4, din (1) rezultă că q 5 ∈ {M4 + 2, M4 + 3} ,
de unde 10p + q 5 ∈ {M4 + 2, M4 + 3} şi nu poate fi pătrat perfect: (M2 + 1)2 =
M4 + 1, (M2 )2 = M4 . Astfel, presupunerea făcută este falsă, iar cele două expresii
nu pot fi simultan pătrate perfecte.

31. Înlocuind m cu 0 ı̂n cea de-a doua relaţie, obţinem: f 2 (n) + f 2 (0) ≤ f (n) ·
n, ∀n ∈ N ⇒ 2 · f 2 (0) ≤ 0 ⇒ f (0) = 0 ⇒ f 2 (n) ≤ f (n) · n, ∀n ∈ N ⇒ f (n) = 0 sau
f (n) ≤ n, ∀n ∈ N ⇒ f (n) ≤ n, ∀n ∈ N(1). Astfel, f (1) ≤ 1.
Dacă f (1) = 0, ı̂nlocuind m cu 1 ı̂n relaţia (i), obţinem: n|(f (n) + n) · f (2), ∀n ∈
N∗ . Din (1) ⇒ f (2) ≤ 2.
Cum: f 2 (n) + f 2 (m) + 2mn ≤≤ (m + n) · f (m + n), ∀m, n ∈ N ⇒ f 2 (n) + 2n ≤
2n
f (n+1)·(n+1), ∀n ∈ N ⇒ 2n ≤ f (n+1)·(n+1), ∀n ∈ N ⇒ f (n+1) ≥ , ∀n ∈
n+1
N ⇒ f (n + 1) > 0, ∀n ∈ N∗ ⇒ f (n) > 0, ∀n ∈ N, n ≥ 2(2).
Astfel, f (2) ∈ {1, 2} .
Dacă f (2) = 1 ⇒ n|f (n) + n, ∀n ∈ N∗
⇒ 2|f (2) + 2 ⇒ 2|3, ceea ce este fals. Atunci, f (2) = 2.
Pentru f (n) = n, n ∈ N∗
(ii)
⇒ f 2 (n) + f 2 (1) + 2n ≤ (n + 1) · f (n + 1)
Soluţiile problemelor propuse 93

n2 + 2n
⇒ f (n + 1) ≥
n+1
n
⇒ f (n + 1) ≥ n +
n+1
f (n+1)∈N (1)
⇒ f (n + 1) ≥ n + 1 ⇒ f (n + 1) = n + 1.
Cum f (2) = 2, din inducţie rezultă:
f1 (0) = f1 (1) = 0, f1 (n) = n, ∀n ∈ N, n ≥ 2 .
Se poate observa că aceasta ı̂ndeplineşte condiţiile din ipoteză.
Dacă f (1) = 1, folosind aceeaşi inducţie ca ı̂n cazul precedent, avem: f (n) =
n ⇒ n2 + 1 + 2n ≤ (n + 1) · f (n + 1)
(1)
⇒ f (n + 1) ≥ n + 1 ⇒ f (n + 1) = n + 1, deci cum f (1) = 1 rezultă:
f2 (n) = n, ∀n ∈ N .

32. Fie n = pα1 1 pα2 2 ...pαk k descompunerea ı̂n factori primi a lui n.
Bergstrom
Atunci, p1 p2 ...pk · φ(n) = pα1 1 pα2 2 ...pαk k · (p1 − 1)(p2 − 1)...(pk − 1) ≥
k ma ≥mg
Bergstrom Y
≥ {[(pi − 1) · αi + 1] · (pi − 1)} ≥
i=1
ma ≥mg k k k k
2− 1
Y p Y Y Y
αi +1 α
≥ (αi + 1) · (pi − 1) · (pi − 1) =
i (αi + 1) · (pi − 1) αi +1 =
i=1 i=1 i=1 i=1
k k
2− α 1+1 1
Y Y
= τ (n) · (pi − 1) i ≥ τ (n) · (pi − 1)2− 2 =
i=1 i=1
k k √
Y 3 p 3 φ(n) Y ( pi − 1)3
= τ (n) · (pi − 1) 2 = τ (n) · ( pi − 1) , de unde ≥ =
i=1
τ (n) i=1
pi
k √ 3
Y pi − 1
√3 p
.
i=1 i

33. Din (i), luând b = 2a rezultă că f (a)a ≤ f (2a), ∀a ∈ N∗ . Cum f (a) + f (b) ≤
2 · f (a)f (b) , f (a)a+1 + f (b) · f (a)a ≤ 2 · f (a)f (b)+a , iar, pentru b par, f (a)a+1 + f (b) ·
94 Soluţiile problemelor propuse

ma≥mg p
f (a)a ≥ f (a)a+1 +f (a)b , deci 2·f (a)f (b)+a ≥ f (a)a+1 +f (a)b ≥ 2· f (a)a+b+1 =
a+b+1 a+b+1
2 · f (a) 2 , f (a)f (b)+a ≥ f (a) 2 , ∀a ∈ N∗ , b > 1, b par.
Să presupunem că există A impar astfel ı̂ncât f (A) > 1. Atunci, f (b) + A ≥
A+b+1
, pentru orice b par, b > 1. Notăm: b = A + 2n + 1.
2 k−1 2
Atunci, f (2k · b) ≥ f (2k−1 · b)2 ·b ≥ f (2k−1 · b)2b ≥ f (2k−2 · b)(2b) ≥ . . . ≥
k−1 k−1 k−1 k
f (2k−k+1 · b)(2b) ≥ f (b)2 ·b , ∀k ∈ N∗ .
= f (2b)(2b)
 2k−1 ·(A+2n+1)k
2n + 2 k−1 k
k
Astfel, f (2 · (A + 2n + 1)) ≥ = (n + 1)2 ·(A+2n+1) (1).
2
S, tim că, ∀k, b ∈ N∗ , există un număr prim p astfel ı̂ncât 2k · b < p < 2k+1 · b
(Conjectura lui Bertrand).
Astfel, din (1)
k−1 ·(A+2n+1)k
f crescatoare 2
⇒ (n + 1)2 ≤ f (2k · (A + 2n + 1)) ≤ f (p) ≤ pp < (2k+1 · (A +
2k+2 ·(A+2n+1)2
2n + 1))2
k+3 k−2
⇒ (2k+1 · (A + 2n + 1))2 > (n + 1)(A+2n+1)
 k−2
A+2n+1 k−2 Bernoulli A + 2n + 1
⇒ (2 k+1
· (A + 2n + 1)) 32
> (n + 1)( 2 )
≥ n· +1 >
2
 k−2
A + 2n + 1

2
(A + 2n + 1)k−34
⇒1>n· , ∀k, n ∈ N∗ .
233k+30
Considerând k = 34, n = 233·35+30 , avem că: 1 > 233·35+30−33·34−30 = 233 , contradicţie.
Presupunerea este falsă, deci nu există A impar astfel ı̂ncât f (A) > 1. Cum
f (2n + 1) = 1, ∀n ∈ N ⇒ f (2n) = 1, ∀n ∈ N∗ (funcţie crescătoare), deci
f (a) = 1, ∀a ∈ N∗ este unica funcţie ce ı̂ndeplineşte condiţiile date.



34. Notăm P (a, b) : f (a)b ≤ a2 f (b) , Q(a, b) : f (a) · b + f (b) · a|(a + b)2 · ab.
P (1, 1) : f (1) ≤ 1, deci f (1) = 1.
Q(a, a) : 2a · f (a)|4 · a4 , deci f (a)|2 · a3 .
P (a, 1) : f (a) ≤ a2 .
Soluţiile problemelor propuse 95

Cum f (2)|16, f (2) ≤ 4, f (2) ∈ {1, 2, 4} .


i) f (2) = 1. Din P (a, 2) : f (a)2 ≤ a2 , deci f (a) ≤ a, ∀a ∈ N∗ .P (a, b) : f (a)b ≤
√ √
√2 2
2 f (b)
a ≤ a2 b ⇒ f (a) ≤ a b , ∀a, b ∈ N∗ . Pentru b → ∞, √ → 0, deci, ∀a ∈
b
∗ 0 ∗
N , f (a) ≤ a = 1, deci f1 (a) = 1, ∀a ∈ N .
Proba: P (a, b) : 1 ≤ a2 , Q(a, b) : a + b|(a + b)2 · ab.
√ √
ii) f (2) = 2. Din P (a, 2) : f (a)2 ≤ a2 2 , deci f (a) ≤ a 2√, ∀a ∈ N∗ .P (a, b) :
√√ √ √
2 b 2
2 2−2 2−2
b
f (a) ≤ a =a 2·b 2
⇒ f (a) ≤ a 2·b 2
, ∀a, b ∈ N∗ . Cum < 0, pentru
√ 2
2−2
b → ∞, b 2 → 0, deci, ∀a ∈ N∗ , f (a) ≤ a0 = 1, dar f (2) = 2 > 1, contradicţie.

iii) f (2) = 4. Din P (2, b) : 4b ≤ 22 f (b)

⇒ 22b ≤ 22 f (b) , ∀b ∈ N∗
p
⇒ 2b ≤ 2 f (b), ∀b ∈ N∗
p
⇒ b ≤ f (b), ∀b ∈ N∗
⇒ b2 ≤ f (b), ∀b ∈ N∗ ,
dar cum f (b) ≥ b2 , ∀b ∈ N∗
⇒ f2 (a) = a2 , ∀a ∈ N∗ .
Proba: P (a, b) : a2b ≤ a2b , Q(a, b) : ab(a + b)|(a + b)2 · ab.

35. Presupunem că există p0 , un număr prim pentru care f (p0 ) ̸= p0 . Atunci, ı̂n
relaţia din enunţ, pe care o vom nota cu (1), vom considera p = p0 şi q = f (p0 ).
Astfel, există x ∈ N astfel ı̂ncât f 2 (p0 ) + 2f (p0 )f (p0 ) + f 2 (f (p0 )) = x2
⇒ 3f 2 (p0 ) = x2 − f 2 (f (p0 )) = (x − f (f (p0 ))(x + f (f (p0 ))).
Dacă f (p0 ) ≥ 5 şi gcd(x, f (f (p0 ))) = 1, atunci (x − f (f (p0 ))(x + f (f (p0 ))) este
impar,
gcd(x + f (f (p0 )), x − f (f (p0 ))) = gcd(x + f (f (p0 )),
2f (f (p0 ))) = gcd(x + f (f (p0 )),
f (f (p0 ))) = gcd(x, f (f (p0 ))) = 1,
deci există 2 posibilităţi: sau x + f (f (p0 )) = f 2 (p0 ), x − f (f (p0 )) = 3, sau x +
96 Soluţiile problemelor propuse

f (f (p0 )) = 3f 2 (p0 ), x − f (f (p0 )) = 1, deci 2f (f (p0 )) =f 2 (p0 ) − 3 sau 2f (f (p0 )) =


f 2 (p0 ) − 3 3f 2 (p0 ) − 1 f 2 (p0 ) − 3
3f 2 (p0 )−1, astfel f (f (p0 )) ∈ , ⇒ f (f (p0 )) ≥ =
2 2 2
f 2 (p0 ) − 2f (p0 ) + 1 + 2f (p0 ) + 4 − 8 (f (p0 ) − 1)2 (f (p0 ) − 1)2
 
= +2+f (p0 )−4 ≥ +
2 2 2
(f (p0 ) − 1)2
 
2+5−4> + 2 ≥ f (f (p0 )) ⇒ f (f (p0 )) > f (f (p0 )), contradicţie.
2
Astfel, dacă f (p0 ) ≥ 5, atunci gcd(x, f (f (p0 ))) > 1; cum f (f (p0 )) este prim,
rezultă că există k ∈ N astfel ı̂ncât x = k · f (f (p0 )), deci egalitatea devine:
3f 2 (p0 ) = (kf (f (p0 )) − f (f (p0 )))(kf (f (p0 )) + f (f (p0 ))) ⇒ 3f 2 (p0 ) = (k + 1)(k − 1) ·
f 2 (f (p0 )). Cum f (p0 ) ≥ 5, f (p0 ) si f (f (p0 )) sunt prime, iar f 2 (f (p0 ))|3f 2 (p0 ), dacă
gcd(f (f (p0 )), f (p0 )) = 1, atunci f 2 (f (p0 ))|3, nu convine, deci gcd(f (f (p0 )), f (p0 )) >
1 şi deci f (f (p0 )) = f (p0 ). Din injectivitate ⇒ f (p0 ) = p0 , ı̂n contradicţie cu
alegerea numărului p0 .
Astfel, dacă există p0 prim astfel ı̂ncât f (p0 ) ̸= p0 , atunci f (p0 ) ∈ {2, 3}. Fie p, q
numere prime astfel ı̂ncât f (p) = 2, f (q) = 3. Fie r un număr prim oarecare,
r ̸= p, r =
̸ q. Atunci, f (r) = r.
 r2 + 2rq + 9 = l2
Atunci, ⇒ 2r(q − p) + 5 = l2 − m2 .
r2 + 2rp + 4 = m2
Pentru orice număr n ∈ N, n2 = M4 sau n2 = M4 +1. Astfel, dacă 2|q−p, l2 −m2 =
M4 + 5 = M4 + 1, deci l2 = M4 + 1, m2 = M4 , astfel 2|r2 + 2rp + 4, ∀r prim,
r ̸= p, r ̸= q, deci orice număr prim ı̂n afară de p şi q este par. Cum există
o infinitate de numere prime, dintre care doar unul este par, rezultă că apare o
contradicţie. Astfel, q şi p au parităţi diferite, p = 2 sau q = 2.
Dacă q = 2, r2 + 4r + 9 este un pătrat perfect pentru orice r prim, r ̸= 2, r ̸= p.
Pentru r = 5, r2 + 4r + 9 = 25 + 20 + 9 = 54, care nu este un pătrat perfect. Pentru
r = 3, r2 + 4r + 9 = 9 + 12 + 9 = 30, care de asemenea nu este un pătrat perfect.
Cum p nu poate fi ı̂n acelaşi timp şi 5, şi 3, rezultă că cel puţin unul dintre aceste
2 cazuri este un contraexemplu, deci q ̸= 2.
Astfel, f (2) = 2. Cum f este injectivă, iar f (r) = r pentru orice număr prim
Soluţiile problemelor propuse 97

r ̸= q, dacă q ̸= 3, atunci f (3) = 3 = f (q), ceea ce contrazice injectivitatea


funcţiei ⇒ q = 3 ⇒ f (3) = 3, deci f (p) = p, ∀p ∈ P .
(p − 1)2
   2 
p − 2p + 1
Proba: p ≤ + 2, ∀p ∈ P ⇔ p ≤ + 2, ∀p ∈ P ⇔ 2 ≤ 2
2 2
p2 − 2p + 5
şi p ≤ , ∀p ∈ P, p ≥ 3 ⇔ 2p ≤ p2 − 2p + 5, ∀p ∈ P, p ≥ 3 ⇔ 4p ≤
2
p2 + 5, ∀p ∈ P, p ≥ 3 ⇔ 12 ≤ 9 + 5 şi p(p − 4) + 5 ≥ 0, ∀p ∈ P, p ≥ 5, ceea ce este
adevărat.
Totodată, f 2 (p) + 2f (p)q + f 2 (q) = p2 + 2pq + q 2 = (p + q)2 , care este un pătrat
perfect, ∀p, q ∈ P, p ̸= q.
Index

Algoritmul lui Euclid, 17 grup, 8-13, 21-24, 29-30


clase de resturi, 29-30 grup multiplicativ, 13, 30
congruenţe, 27-28 - [3], [26] subgrup, 21-24
Teorema lui Lagrange, 28 semigrup, 7-8, 22-24
ordinul gaussian, 28 monoid, 8-9, 11, 22, 29-30
rădăcini primitive, 28 Teorema lui Cauchy, 23 - [6]
Teorema chineză a resturilor, 28 Teorema lui Lagrange, 23 - [6]
Conjectura lui Catalan, 36 - [14] Identitatea lui Bezout, 18
Conjectura lui Collatz, 20 - [22] inel, 11-12, 22-24, 30
corp, 13, 22-23, 30 inel integru, 13
Teorema lui Wedderburn, 22 lege de compoziţie, 7-9
ecuaţii de tip Pell - [18] Lifting the Exponent Lemma, 37 - [7],[9]
Ecuaţia Pell, 39 morfism, 21-22, 23-24 - [5]
Ecuaţia Pell negativă, 39-40 omomorfism, 21
ecuaţii diofantiene - [18] izomorfism, 21, 23-24
ecuaţii diofantiene liniare, 33 endomorfism, 21, 23
triplete pitagoreice, 34 automorfism, 21-23
ecuaţii diofantiene pătratice, 35 Secvenţe Farey, 43 - [3]
ecuaţii diofantiene de grad superior, 36 Secvenţe Lucas, 45-46 - [27]
Extindere Cantor, 20 Numere Lucas, 45 - [28]
funcţii aritmetice Numere Fibonacci, 46 - [29]
funcţii multiplicative, 25-26 Formulele lui Binet, 46 - [28], [29]
Funcţia lui Euler, 25 Secţiunea de aur, 46 - [30]
Funcţia lui Mobius, 27 Simbolul lui Legendre, 41 - [3]

98
99

rest pătratic, 41
Criteriul lui Euler, 41
Lema lui Gauss, 42
Legea de reciprocitate a lui Gauss, 42
Teoremele lui Fermat - [3]
Mica Teoremă a lui Fermat, 31
Marea Teoremă a lui Fermat, 36
Teorema lui Wilson, 31 - [3]
Teorema lui Wolstenholme, 47-48 - [16]
Teorema lui Zsigmondy, 47 - [15]
100 Soluţiile problemelor propuse

Bibliografie

[1] Ana Maria Bâcă, Scurtă introducere ı̂n Teoria numerelor, 2020.
[2] Marius Burtea, Georgeta Burtea, Matematică. Manual pentru clasa a XII -a. M1,
Editura Carminis, 2007.
[3] Magdaş Camelia, Moldovan Dorin, Introducere ı̂n Teoria Numerelor, Editura
GIL, 2018
[4] Florentina Boboc, Dana Piciu, Dumitru Buşneag (coordonator), Aritmetică şi
teoria numerelor, 1999
[5] Ion D. Ion, Gabriela Streinu-Cercel, Adrian P. Ghioca, Neculai I. Nediţă, Eu-
gen Câmpu, Nicolae Angelescu, Boris Singer, Romeo Ilie, MATEMATICĂ. Manual
pentru clasa a XII-a. M2, Editura Sigma, 2007
[6] artofproblemsolving.com, Lifting The Exponent Lemma (LTE)
[7] taharut.org, Lifting The Exponent Lemma (LTE)
[8] basarab.ro, Caleidoscop matematic nr. 2
[9] mathworld.wolfram.com, Catalan’s Conjecture
[10] Mathematical Excalibur, Mathematical Excalibur, Vol. 16, No. 4
[11] mathworld.wolfram.com, Wolstenholme’s Theorem
[12] Titu Andreescu, Dorin Andrica, O introducere ı̂n studiul ecuaţiilor diofantiene,
Editura GIL, 2002
[13] Mathematical Excalibur, Mathematical Excalibur, Vol. 19, No. 3
[14] ijpam.eu, The relation among Euler’s phi function, tau function, and sigma function
[15] Kenneth H. Rosen, Elementary Number Theory and Its Application, Library of
Congress Cataloging in Publication Data, 1984
[16] Factorialul unui număr natural nenul
[17] Rădăcini primitive modulo n
[18] Laurenţiu Ploscaru, Ordinul unui numar modulo n
Soluţiile problemelor propuse 101

[19] mathworld.wolfram.com, Lucas Sequence


[20] mathworld.wolfram.com, Lucas Number
[21] mathworld.wolfram.com, Fibonacci Number
[22] mathworld.wolfram.com, Golden Ratio

S-ar putea să vă placă și